VetPrep NAVLE Flashcards

1
Q

An 8-year old Thoroughbred gelding was presented for acute onset of trembling, excessive recumbency, constant shifting of body weight while standing, and muscle atrophy. The horse is housed in a dry lot with no pasture availability. Diet consists of moderate quality grass hay and sweet feed. The CBC is normal with abnormalities on biochemistry analysis including mild elevation in creatine kinase (CK) and aspartate aminotransferase (AST). Cerebrospinal fluid (CSF) analysis shows no remarkable findings. Based on this information, what disease process do you suspect?

Equine Herpes Myeloencephalopathy (EHV-1)
Equine Motor Neuron Disease (EMND)
Equine Protozoal Myeloencephalitis (EPM, Sarcocystis neurona)
Verminous encephalomyelitis (Halicephalobus deletrix)
Cervical Vertebral Malformation (Wobbler Syndrome)

A

Answer: Equine Motor Neuron Disease (EMND)

The correct answer is EMND. This is a neurologic disease associated with dietary vitamin E deficiency which is believed to contribute to oxidative damage to the CNS. However, the precise pathophysiology is unknown.

Retrospective studies have noted that horses with EMND are commonly housed on dry lots with little to no availability to pasture or good quality hay (perhaps contributing to low blood vitamin E concentrations). The clinical signs in this question are classic for EMND whereas horses afflicted with the other options do not typically demonstrate trembling or shifting of body weight.

How well did you know this?
1
Not at all
2
3
4
5
Perfectly
2
Q

Dr. Jones is examining Snuggles, an 8-year-old male castrated Domestic Shorthair cat. Snuggles’ owner is Anna, who is a physician. Anna noted that Snuggles wasn’t eating well two days ago. Today Snuggles began vocalizing in the litterbox. Snuggles lives with two other cats, so Anna is not sure whether he is urinating successfully. On palpation of the abdomen, Dr. Jones feels a large, turgid urinary bladder. Which of the following is the most appropriate way for Dr. Jones to begin a discussion about urethral obstruction in cats?

  • Do you already know about urethral obstruction in cats?
  • Most clients aren’t familiar with urethral obstruction in cats.
  • I’m sure you know all about urethral obstruction in cats.
  • How familiar are you with urethral obstruction in cats?
A

Answer: How familiar are you with urethral obstruction in cats?

Use of a question to gauge the client’s current knowledge base is recommended. This option does not suggest an assumption of the client’s level of knowledge. This option is posed as an open question, which maximizes the amount of information that the client is likely to share. The use of open-ended questions is encouraged to improve client communication. Open-ended questions allow the client to provide a comprehensive answer, which gives the most information to the medical team. Additionally, this option does not lead the client toward a particular answer. Questions beginning with “do” are closed questions, or questions for which there is only one answer (yes or no).

Closed questions limit the information the client is being invited to provide. Both statement options assume the level of knowledge the client possesses and should therefore be avoided.

Source: Handbook of Veterinary Communication Skills. Edited by Carol Gray and Jenny Moffett. Wiley-Blackwell, 2010. Page 14 and 31-33.

How well did you know this?
1
Not at all
2
3
4
5
Perfectly
3
Q

A cat is having a dental procedure and you notice some debris in the ears. You flush the ear canals gently and clean away the debris. Upon recovery from anesthesia, you notice the cat has a protrusion of the right nictitating membrane and the right pupil is miotic. The left eye appears normal. What is the most likely cause of these clinical symptoms?

  • Trauma to spinal cord between T1-T4 while under anesthesia
  • Vascular accident from anesthesia induced hypertension
  • Rupture of tympanic membrane
  • Inner ear polyp
A

Answer: Rupture of tympanic membrane

Horner’s syndrome occurs from a disruption of the sympathetic innervation to the eye. In addition to the clinical signs mentioned, a droopy upper eyelid, enophthalmos, nystagmus, and sometimes a head tilt can occur. Because this cat was normal before the procedure, the likely cause of the Horner’s syndrome is a tympanic membrane (eardrum) rupture or trauma that occurred during the ear flush. This is because the sympathetic innervation to the eye runs through the middle ear. An inner ear polyp could also cause this syndrome; however, because it came on acutely after an ear flush, a ruptured ear drum is most likely.

Anesthesia typically causes hypotension, not hypertension, so a vascular accident is very unlikely to occur in a young healthy cat.

A spinal cord lesion at 11-14 could also cause this syndrome due to the sympathetic preganglionic cell bodies located in this area, but a significant trauma would have to occur for this to result in Horner’s syndrome, and it is less likely in this anesthetized cat.

How well did you know this?
1
Not at all
2
3
4
5
Perfectly
4
Q

You go to a dairy which is having trouble with sudden death in calves and decreased reproductive efficiency in cows. Which plant toxin should you have on your differential list?

  • Tansy ragwort
  • Moldy sweet clover
  • Larkspur
  • St John’s-wort
  • Gossypol
A

Answer: Gossypol

The correct answer is gossypol (found in cotton seed in varying amounts) toxicity.

Gossypol is a cardiotoxin and can kill calves less than 4 months of age. In adults, if fed at high levels, it usually causes sterility in bulls and decreased conception in cows.

Moldy sweet clover inhibits vitamin K and causes a coagulopathy.

St. John’s-wort contains hypericin and results in photosensitivity.

Tansy ragwort is a pyrrolizidine alkaloid and causes liver disease.

Larkspur causes general nonspecific signs such as salivation, bloat, muscle fasciculation, collapse, staggering, and death.

How well did you know this?
1
Not at all
2
3
4
5
Perfectly
5
Q

You are doing a summer externship in South America and performing physical exams on a variety of animals. You are performing a fundic exam on the eye of a horse and note what appears to be a worm migrating through the conjunctiva. Because of the location and appearance of this parasite, you suspect this is which of the following?

A

Answer: Thelazia

Thelazia is a genus of nematode worms (eyeworms) which are found in the ocular tissues.

Adults are usually found in the eyelids, tear glands, tear ducts, or the nictitating membrane. They may be found in the eyeball itself under the conjunctiva or in the vitreous. Thelazia are transmitted by Diptera (flies) which do not bite but feed on tears.

Toxocara, which causes ocular larval migrans, usually causes granulomas which may be seen in the retina and appear more circular.

How well did you know this?
1
Not at all
2
3
4
5
Perfectly
6
Q

Answer: Thelazia

Thelazia is a genus of nematode worms (eyeworms) which are found in the ocular tissues.

Adults are usually found in the eyelids, tear glands, tear ducts, or the nictitating membrane. They may be found in the eyeball itself under the conjunctiva or in the vitreous. Thelazia are transmitted by Diptera (flies) which do not bite but feed on tears.

Toxocara, which causes ocular larval migrans, usually causes granulomas which may be seen in the retina and appear more circular.

A

Answer: Yellow star thistle.

Yellow star thistle is Centaurea solstitialis. In some horses which ingest large quantities, it can cause nigropallidal encephalomalacia, which is loss of the globus pallidus and substantia nigra, and extrapyramidal nuclei, which control muscles of prehension. Loss of these nuclei causes tremors and dystonia of the lip muscles and tongue.

Centaurea repens (Russian knapweed) can also cause similar lesions.

How well did you know this?
1
Not at all
2
3
4
5
Perfectly
7
Q

When formulating late gestation anionic diets for dairy cows to help prevent hypocalcemia in the last 2 to 3 weeks prior to calving, what formula is used?

  • DCAD = (Na - Cl) + (K - S)
  • DCAD = (Na - K) + (P - S)
  • DCAD = (Na + S) - (K + Cl)
  • DCAD = (Na + K) - (Cl + S)
  • DCAD = (Na + Cl) - (K + S)
A

Answer: DCAD = (Na + K) - (Cl + S)

DCAD stands for Dietary Cation Anion Difference. Na=sodium, K=potassium, Cl=chloride, and S=sulfur. These are the 4 most important strong ions to be considered. When the diet is optimal the urine pH of Holstein cows should range between 6.2 and 6.8 for cows on the ration.

How well did you know this?
1
Not at all
2
3
4
5
Perfectly
8
Q

Which of the following is true about porcine reproductive and respiratory syndrome (PRRS)?

  • Treatment of the herd involves prolonged, aggressive antibiotic therapy added to the food
  • Infected boars may be infertile due to poor sperm quality
  • Clinical signs are seen only in pigs at breeding age
  • The disease is caused by a herpes virus
A

Answer: Infected boars may be infertile due to poor sperm quality

The correct answer is infected boars may be infertile due to poor sperm quality. PRRS is caused by an arterivirus and affects pigs of all ages. Affected neonatal pigs experience diarrhea and recurrent fevers. Weaned pigs may become anorexic, develop a cough, dyspnea (or “thumps”), and purple ear tips and tails due to vasculitis.
Gestating sows have reproductive failure in the form of abortions, mummies, and early embryonic death. Boars may be infertile due to interference with spermatogenesis. Treatment of a herd may involve depopulating the herd, or closing the herd for several months and following titers.

How well did you know this?
1
Not at all
2
3
4
5
Perfectly
9
Q

A 10-year old male castrated schnauzer presents laterally recumbent and minimally responsive. The owners just returned from vacation and his pet sitter noted he seemed normal yesterday. His temperature is 97 F (36.1 C), heart rate is 50 beats per minute (normal 60-160 beats per minute), and respiratory rate is 50 breaths per minute (normal 10-30 breaths per minute). You determine he is approximately 8% dehydrated based on skin turgor. Systolic blood pressure is 80 mmHg (normal 100-140 mmHg). He groans when his abdomen is palpated. You find a small pocket of ascites on a brief ultrasound exam. Cytology of the fluid shows a moderate amount of neutrophils with no obvious bacteria. An in-house chemistry panel is within normal limits (glucose 145, sodium 145, potassium 4.10, lactate 0.8). The ascites glucose is 100 and lactate is 2.8. What is your recommendation to the owners?
Start active warming and give a dose of atropine
Crystalloid fluid therapy and antibiotics
Surgical exploratory laparotomy
Colloid fluid therapy and antibiotics

A

Anwer: Surgical exploratory laparotomy

This dog has a septic abdomen and requires surgical intervention. Sepsis is suspected if the glucose in the ascites is at least 20mg/dL less than the peripheral blood. A blood lactate level of 2.0mmol/L lower than the ascites lactate level is also highly suggestive of a septic abdomen. The presence of intracellular bacteria would also be diagnostic for a septic abdomen. The hypotension, bradycardia, and hypothermia can be attributed to the sepsis as well. An abdominal ultrasound may help to determine the source. Rehydration and antibiotics are necessary components of treatment but ultimately surgical exploration is necessary.

How well did you know this?
1
Not at all
2
3
4
5
Perfectly
10
Q

You diagnose this 15-year old mare in the picture with a mild, acute laminitis. Which of the following can be used as treatment for this horse?

  • Phenoxybenzamine
  • Prednisone
  • Application of horse shoes
  • Trimethoprim sulfa
A

Answer: Phenoxybenzamine

The correct answer is phenoxybenzamine.

Phenoxybenzamine is an alpha-adrenergic antagonist promoting vasodilation and restoration of blood flow to the digits. Prednisone is contraindicated in laminitis because corticosteroids are believed to induce the condition. Antibiotics are not indicated unless a secondary bacterial infection develops. Application of a horse shoe would not help and would be very painful in an already sensitive and painful condition. Other medications used to restore blood flow to the digits include acepromazine, isoxsuprine hydrochloride, dimethylsulfoxide (DMSO), heparin, and nitroglycerine.

How well did you know this?
1
Not at all
2
3
4
5
Perfectly
11
Q

Every time an owner shears his sheep, 10% develop large abscesses in their lymph nodes several weeks or even months after shearing. What should the owner do to control/prevent this problem the next time he shears his sheep?
Disinfect shears before shearing and between shearing sheep
Pretreat all animals by dipping in a solution of dilute Betadine (an aqueous iodine compound) in a dip tank
To eliminate carriers of the disease from the flock, treat all affected animals with long acting tetracycline when you lance the abscesses later
Develop a closed flock in which all replacements come from within the flock

A

Answer: Disinfect shears before shearing and between shearing sheep

The correct answer is to disinfect shears before beginning and between shearing sheep. The sheep are most likely being infected with caseous lymphadenitis (also commonly called boils) which is caused by Corynebacterium pseudotuberculosis. This organism is highly contagious and commonly transmitted at shearing when nicks and cuts occur. When you lance these abscesses, make sure to clean up the area thoroughly and decontaminate everything to avoid spreading the disease.The rancher should also consider vaccination against caseous lymphadenitis.Dipping in Betadine would be prohibitively expensive and likely to stain the wool and be ineffective. Neither other choice is useful in controlling caseous lymphadenitis.

How well did you know this?
1
Not at all
2
3
4
5
Perfectly
12
Q

You are called to a small goat dairy to examine three ill does, which are in the final stages of late pregnancy, have stopped eating, and appear to be weak. One has actually died just before you arrived, and you perform a post mortem exam on her. You are amazed to find 5 dead near-term fetuses inside, as well as a pale fatty liver (see photo). Based on this and the appearance of the others, you check their urine and find ketones. What is your diagnosis?
Hypocalcemia
Gossypol toxicity
Hypomagnesemia
Pregnancy toxemia
Hepatotoxicity

A

Answer: Pregnancy toxemia.

Multiple fetuses can become a space-occupying lesion, making the rumen more difficult to fill, just as the doe requires maximum caloric intake. As she tips over into negative energy balance, utilizes her omental fat stores and develops a fatty liver, she becomes keto-acidotic and develops pregnancy toxemia. To be saved, the other affected animals must have IV glucose and an immediate C section, followed by excellent nutritional support.
They may also need calcium, magnesium and other electrolytes.

How well did you know this?
1
Not at all
2
3
4
5
Perfectly
13
Q

Which of these drugs is a neuromuscular blocker when given to a dog?
Naloxone
Neostigmine
Atracurium
Edrophonium
Fentanyl

A

Answer: Atracurium.

The correct answer is atracurium. Atracurium is a non-depolarizing neuromuscular blocker. Other neuromuscular blockers include pancuronium, d-tubocurarine and succinylcholine. Succinylcholine is a depolarizing neuromuscular blocker. Fentanyl is an opioid. Naloxone is an opioid antagonist. Edrophonium and neostigmine are cholinesterase inhibitors used to reverse neuromuscular blockers.

How well did you know this?
1
Not at all
2
3
4
5
Perfectly
14
Q

What species is the principle carrier and vector of Malignant Catarrhal Fever (MCF) virus in North America?
Bison
Horse
Cattle
Sheep
Donkey

A

Answer: Sheep.

MCF virus is ovine herpes virus-2 (OvHV-2) and is carried by 95-99% of sheep in North America which show no symptoms. It is also carried by 75% of domestic goats, 40% of muskox, 37% of bighorn sheep, 25% of pronghorn antelope, 62% of mouflon sheep, and by a small percentage of elk, mule deer, and white tailed deer. Susceptible hosts include cattle, water buffalo, deer, pigs, and bison; bison are the most susceptible.

How well did you know this?
1
Not at all
2
3
4
5
Perfectly
15
Q

What is the primary mode of transmission of West Nile Virus (WNV) between birds?
Via feces
Via placenta
Via mosquitoes
Via Cnemidocoptes mites
Via saliva

A

Answer: via mosquitoes.

Mosquito vectors are the primary mode of transmission for WNV between birds and other hosts (horses, humans). There are infrequent documented cases of the disease being spread by feces or saliva. No mites have been documented to transmit the disease. Birds do not have a placenta.

How well did you know this?
1
Not at all
2
3
4
5
Perfectly
16
Q

A backyard hen presents with a history of being previously active followed by sudden death with no premonitory signs. Gross examination reveals an obese hen with pale, friable liver and broad blood clots attached to the liver capsule and extending into the abdomen. There is no evidence of external trauma. The most likely diagnosis is:
Vitamin K deficiency
Histomonas meleagridis infection
Vitamin E deficiency
Larval ascarid migration
Fatty liver/hemorrhagic syndrome

A

Answer: Fatty liver/hemorrhagic syndrome

Excess lipid accumulation in liver of hens can promote fractures and fatal hepatic hemorrhage.

How well did you know this?
1
Not at all
2
3
4
5
Perfectly
17
Q

Which of the following is the causative agent of fowl cholera?

  • Mycoplasma gallisepticum
  • Chlamydophila psittaci
  • Reticuloendotheliosis virus
  • Pasteurella multocida
A

Answer: Pasteurella multocida

The correct answer is Pasteurella multocida.

The bacterium is a gram negative rod. It causes fever, mucoid discharge from the mouth, diarrhea, petechia, ecchymoses, increased pericardial and peritoneal fluid, and death. The chronic form of the disease usually causes localized disease. Diagnosis is based on identifying the organism from samples in conjunction with clinical signs. Vaccines are available for prevention of the disease. Sulfa antibiotics are used for treatment.

How well did you know this?
1
Not at all
2
3
4
5
Perfectly
18
Q

A 2-year old MN DSH has recently been treated for a urethral obstruction. He went home last week from the hospital on an acidifying canned diet for this condition.
The owner reports he is passing urine in moderate amounts, but he is still straining frequently. You reexamine the cat and find that the bladder is empty on palpation and the wall feels a little thickened. You are confident that the cat has not re-blocked and the cat’s bloodwork shows normal electrolytes and renal values.

Which of the following medications may help the cat with this problem?

  • Amitriptyline
  • Phenoxybenzamine
  • Phenylpropanolamine
  • Methocarbamol
  • Prednisolone
A

Answer: Phenoxybenzamine

This cat is likely suffering from hypertonicity of urethral muscle, which was incited from the recent obstruction and urinary catheter. This can cause spasms, which makes urinating painful and not easily controlled.

Phenoxybenzamine can be used in this case to reduce internal urethral sphincter tone such that the cat may urinate more easily.

Methocarbamol is a muscle relaxant but would not directly help spasms of the urethra.
Prednisolone is not used to help reduce inflammation or spasms in the urethra and may predispose the cat to contracting a urinary infection, especially while his bladder and urethra are compromised.

Phenylpropanolamine is used to treat urinary incontinence from urethral hypotonicity most often in dogs and would be contraindicated in this case.

Amitriptyline is an anti-depressant medication that has been implicated as part of a treatment plan for cats with cystitis, although benefit has never been proven. Because cats with cystitis can flare up during times of stress, the amitriyptyline has been thought to help prevent this. This medication would not work to stop spasms in the urethra.

How well did you know this?
1
Not at all
2
3
4
5
Perfectly
19
Q

Which of the following swine pathogens is known to attach to and disrupt the pulmonary mucociliary apparatus?

  • Mycoplasma hypneumoniae
  • Salmonella choleraesuis
  • Porcine reproductive and respiratory syndrome virus (PRRSv)
  • Actinobacillus pleuropneumoniae
  • Porcine circovirus type 2 (PCV2)
A

Answer: Mycoplasma hyopneumoniae

The correct answer is Mycoplasma hyopneumoniae. The bacteria attach to the pulmonary mucociliary apparatus and thus prevent it from functioning properly. This in turn makes the lungs much more susceptible to other bacterial infections.

How well did you know this?
1
Not at all
2
3
4
5
Perfectly
20
Q

Which of these inhalant anesthetics has the fastest onset of action in the dog?

Desflurane
Isoflurane
Diethyl ether
Halothane

A

Answer: Desflurane

The correct answer is desflurane. Speed of action of inhalant anesthetics is most directly related to blood-gas solubility. There is an inverse relationship between rapidity of action and the anesthetic’s solubility in the blood, meaning the lower an inhalant’s solubility, the faster its onset of action. This is expressed as the bloodigas partition coefficient. Desflurane has the lowest bloodigas partition coefficient and therefore, has the fastest time of action. In order from fastest to slowest, the inhalants are (desflurane, NO, sevoflurane, isoflurane, enflurane, halothane, diethyl ether, methoxyflurane).

How well did you know this?
1
Not at all
2
3
4
5
Perfectly
21
Q

A 1-year old male neutered Siamese presents to you with the clinical signs seen in the image. He is an indoor only cat and the owners recently adopted an 8 week old kitten that he is in contact with. His clinical signs were initially mild and unilateral but progressed to these findings three days later. What are the two most likely pathogens affecting this cat and what is the treatment of choice?

  • Feline Calicivirus and Feline Leukemia Virus, topical trifluridine
  • Chlamydophilia felis and Cornybacterium
    Spp., topical prenisone acetate
  • Feline Calicivirus and Chlamydophilia felis, oral clavamox
  • Feline Herpesvirus-1 and Chlamydophila felis, topical tetracycline
  • Mycoplasma felis and Feline Calicivirus, oral amoxicillin
A

Answer: Feline Herpesvirus-1 and Chlamydophila felis, topical tetracycline

The most common cause of bacterial conjunctivitis in feline patients is Chlamydophila felis. This disease cannot be distinguished from feline Herpesvirus-1 based on clinical appearance alone.
Frequently, feline patients with these symptoms are infected with both of these diseases simultaneously. Diagnosis of Chlamydophila felis is made by visualizing typical elementary bodies in the cytoplasm of conjunctival epithelial cells or obtaining a positive fluorescent antibody (FA) test on a conjunctival scraping. The treatment of Chlamydophila felis is topical tetracycline QID for one week post resolution of clinical signs.

How well did you know this?
1
Not at all
2
3
4
5
Perfectly
22
Q

A 10-year-old female spayed DSH presents for dropping food and drooling from the left side. She has no history of trauma but recently has had thick ocular discharge which the owner has had to clean regularly. You perform a neurologic exam and find the following (see image).

The rest of the exam is within normal limits. What do you inform the owner?

  • The recommended treatment is a ventral bulla osteotomy
  • The cause is most likely idiopathic and carries a good prognosis
  • Treatment involves corticosteroid
    administration but prognosis for recovery is still guarded
  • The other side may start to show the same signs within a few weeks to months
A

Answer: The other side may start to show the same signs within a few weeks to months

This patient has the classic finding associated with unilateral facial nerve paresis.

Unfortunately, it is most commonly idiopathic; however in cats it can sometimes be caused by things such as nasopharyngeal polyps, neoplasia and trauma.

Prognosis for recovery is guarded and the clinical signs are typically permanent. The unaffected facial nerve can become affected as well.

Treatment for idiopathic disease is supportive including eye lubrication and management of corneal ulcers. Drooling usually resolves over several weeks. Steroids have been used in humans with Bell’s palsy but it has not been shown to be effective in animals.

Otitis media-interna and chronic ear disease can make animals at higher risk for nerve paralysis and a CT of the bulla is recommended rather than radiographs. Bulla osteotomy may be
recommended for animals with middle ear disease or those prone to chronic otitis media-interna. In dogs, facial paralysis has been seen with hypothyroidism.

How well did you know this?
1
Not at all
2
3
4
5
Perfectly
23
Q

A client brings in her dog that recently ingested rodenticide. You check the box and see that the active ingredient is cholecalciferol. The dog appears clinically normal right now. What would you be concerned might happen if you do nothing at this time?

  • Development of organ mineralization
  • Development of coagulopathy
  • Development of liver failure
  • Development of intestinal perforation
  • Development of neurologic signs
A

Answer: Development of organ mineralization

Explanation
The correct answer is development of tissue mineralization. Cholecalciferol (Vitamin D3) acts by becoming converted to calcitriol and increases calcium and phosphorus levels leading to mineralization of organs, especially the kidneys.

How well did you know this?
1
Not at all
2
3
4
5
Perfectly
24
Q

A 10 year old female spayed Dachshund presents for further evaluation of progressive lameness in the hind.

The owner notes that she has begun intermittently scuffing her hind limbs, has somewhat of a weak or drunken gait in the hind as she walks, and is having trouble rising. Given the answer choices below, what is the best treatment?

  • Femoral head ostectomy
  • Total hip replacement bilaterally
  • Prolonged antibiotic therapy with clindamycin
  • Dorsal laminectomy
A

Answer: Dorsal laminectomy

Explanation
This patient’s clinical signs are consistent with neurological deficits. Although you do not have all of the information necessary to reach a definitive diagnosis, you should have been able to deduce that this patient probably needs a dorsal laminectomy to relieve disc compression.
In order to definitively diagnose the condition you would want to proceed with a complete exam, basic bloodwork, and a CT or MRI of the lumbosacral region. Although this patient does exhibit evidence of hip dysplasia radiographically, the clinical signs are not characteristic of a dog with pain secondary to hip dysplasia. There is no indication of osteomyelitis on these radiographs and the clinical signs are not necessarily suggestive of osteomyelitis.

It is important not to skip ahead to image interpretation without carefully reading the question. Doing so in this case would likely lead you to select the incorrect answer.

Radiographs provided by Adrien Hesperian, DVM, MS, DACVR and University of Tennessee College of Veterinary Medicine

How well did you know this?
1
Not at all
2
3
4
5
Perfectly
25
Q

A cat with a Cryptococcus infection and liver failure should not be treated with ketoconazole for which of the following reasons?

  • It causes hypoadrenocorticism in cats
  • It is nephrotoxic to cats
  • diarrhea in cats
  • It is hepatotoxic and causes vomiting
  • It is not effective in treating Cryptococcus
A

Answer: It is hepatotoxic and causes vomiting

Explanation
The correct answer is it is hepatotoxic and causes vomiting and diarrhea in cats.

Ketoconazole is an azole antifungal. It is effective against Cryptococcus, but it causes severe Gl upset and anorexia, particularly in cats. It is also hepatotoxic, making it a poor choice for the cat described in the question.

How well did you know this?
1
Not at all
2
3
4
5
Perfectly
26
Q

What is the primary goal in the initial treatment of diabetic ketoacidotic dogs?

  • Getting the animal to eat
  • Controlling obesity or concurrent disorders
    causing insulin resistance
  • Lowering the elevated blood glucose
  • Correction of acidosis, electrolyte abnormalities, and hyperosmolality
A

Answer: Correction of acidosis, electrolyte abnormalities, and hyperosmolality

The correct answer is correction of acidosis, electrolyte abnormalities, and hyperosmolality.

Initial treatment of DKA patients is aimed at correcting acidosis by administering bicarbonate (if the total bicarbonate is markedly low), correcting electrolyte abnormalities with IV fluids, potassium and phosphorus supplementation, and correction of hyperosmolality. Regulating blood glucose, getting the animal to eat, controlling obesity and concurrent disease should be addressed after immediate life-threatening issues are resolved.

How well did you know this?
1
Not at all
2
3
4
5
Perfectly
27
Q

A dog presents to you after being struck in the head with a baseball bat. The dog is obtunded, but the rest of the physical exam is normal. Which of the following factors should you try to achieve in this patient to minimize intracranial pressure?

  • High Pa02
  • Low mean arterial pressure
  • High PaCO2
  • High cerebral metabolic rate
A

Answer: High Pa02

Explanation
The correct answer is high PaO2.

PaCO2, blood pressure, PaO2, cerebral metabolism, and certain drugs can affect cerebral blood volume and intracranial pressure. In head trauma patients, you want to prevent an increase in intracranial pressure by keeping cerebral blood flow relatively low without causing hypoxia. To keep cerebral blood flow in the low-normal range, the cerebral metabolic rate and PaCO2 should be kept low.

Factors that would cause the cerebral metabolic rate to go up include fever, seizures, pain, etc.
Mean arterial pressure should be maintained in the normal range to prevent hypotension, hypoxia, and cerebral ischemia.

How well did you know this?
1
Not at all
2
3
4
5
Perfectly
28
Q

You have a 6-year old female spayed Golden Retriever patient with inappetence and vomiting. You diagnose diabetic ketoacidosis based on a blood glucose of 641 mg/dL (76-119 mg/dL), 3+ glucose (normal- negative) and 3+ ketones (normal- negative) in the urine, and blood pH of 7.12 (7.35-7.45). You would like to treat with the shortest acting, most potent insulin type. Which of the following would you choose?

  • Ultralente
  • NPH
  • PZI
  • Regular
  • Lente
A

Answer: Regular.

Listed from shortest acting and most potent to longest acting and least potent: Regular, NPH, Lente, PZI, Ultralente. The treatment of DKA involves IV fluids and regular insulin, given either by CRI or the intermittent IM dosing technique.

Regular insulin is continued until the patient is eating well and is hydrated, and can be switched to a longer-acting insulin, to be given SQ at home.

How well did you know this?
1
Not at all
2
3
4
5
Perfectly
29
Q

Dr. Jones has just read a new study regarding anthelmintic use in puppies. The study suggests that the practice should change its standard operating procedure regarding fecal parasite testing and anthelmintic administration schedules for puppies. The current intestinal parasite management protocol for puppies has been in place for over a decade, and Dr.Jones recognizes that he is likely to encounter resistance to the necessary changes. Dr. Jones’ staff consists of one Licensed Veterinary Technician and four unlicensed Veterinary Assistants. Which of the following options is the most appropriate method for sharing the results of the study with the veterinary staff?

Create a presentation for the staff which focuses on the descriptive statistics and discussion portions of the study.
Prepare a brief presentation for the staff, create visual aids, and provide an informal written summary of the study.
Instruct the staff members to read the study, tell the staff to come to you with any questions, and give the staff a test on the material.
Give the staff members a copy of the study and have the staff prepare a presentation on the findings.

A

Answer: Prepare a brief presentation for the staff, create visual aids, and provide an informal written summary of the study

Explanation
Using a combination of communication formats - verbal, visual and written - to explain concepts is ideal.

Informal language should be used to communicate with lay staff. Highly technical terminology should be avoided. While having the staff read the study independently might sound ideal, some individuals do not possess the knowledge of statistics and medical terminology required to fully understand a study. Requiring staff to take test or create a presentation on unfamiliar material might create anxiety. While creating a presentation for the staff is a good idea, focusing on the descriptive statistics is not recommended. Highly technical or advanced terminology should be avoided.

Source: Handbook of Veterinary Communication Skills. Edited by Carol Gray and Jenny Moffett. Wiley-Blackwell, 2010. Page 127-148.

The most appropriate method for Dr. Jones to share the study results with the veterinary staff is to prepare a brief presentation, create visual aids, and provide an informal written summary of the study. This approach combines verbal, visual, and written communication, ensuring the information is accessible without overwhelming the staff with technical details. Avoiding highly technical terminology and using informal language will help in effectively conveying the necessary changes to the team.

How well did you know this?
1
Not at all
2
3
4
5
Perfectly
30
Q

A new client has presented a ten-year-old female spayed domestic shorthair cat for evaluation. The client reports that the patient has been losing weight and urinating outside of her litterbox for the past six months. The patient had been eating well until three days ago, when she stopped eating and began vomiting. Physical examination reveals that the patient is emaciated and severely dehydrated. A glucometer reading shows that the blood glucose is too high to register. The client reports that they do not have the financial resources for further testing or treatment of a major illness, and they wish to proceed with humane euthanasia. What is the most appropriate next step?

  • Escort the client and patient to a quiet, comfortable space prior to humane euthanasia.
  • Discuss options for handling the patient’s remains prior to humane euthanasia.
  • Confirm client ownership of the patient prior to humane euthanasia.
  • Obtain verbal consent from the client prior to humane euthanasia.
A

Answer: Confirm client ownership of the patient prior to humane euthanasia

Explanation
Confirming client ownership of the patient is the most appropriate next step since the client and patient are not previously known to the clinic/ veterinarian. While obtaining client consent is important prior to humane euthanasia, verbal consent is less appropriate than written consent when the client and patient are not previously known to the clinic/veterinarian. Discussing options for handling the patient’s remains and escorting the client to a comfortable, quiet area are important prior to humane euthanasia, however confirming client ownership should be completed first.

Source: Handbook of Veterinary Communication Skills. Edited by Carol Gray and Jenny Moffett. Wiley-Blackwell, 2010. Pages 55-56.

The most appropriate next step is to confirm client ownership of the patient before proceeding with humane euthanasia, especially since the client and patient are new to the clinic. While obtaining written consent and discussing options for the patient’s remains are important, verifying ownership should be prioritized to ensure proper and ethical procedures are followed.

How well did you know this?
1
Not at all
2
3
4
5
Perfectly
31
Q

Annabelle is a twelve-year-old Labrador Retriever with progressive hindlimb pain and weakness. The pet is unable to stand without assistance and walks with a stiff gait when assisted to a standing position. The owner does not wish to pursue additional testing or treatment. Today a euthanasia appointment was scheduled by the owner. However, upon arrival the owner expresses doubt about whether euthanasia is the right thing to do. Which is the most appropriate response to the owner’s concerns?

  • Explain that feeling doubt regarding a euthanasia decision is unusual and indicates that the owner is not yet ready to make a decision regarding euthanasia.
  • Assure the owner that feeling doubt regarding a euthanasia decision is normal and encourage the client to complete a quality-of-life score for the patient.
  • Discuss the reasons that the owner is experiencing doubt about a euthanasia decision and assure the owner that euthanasia is the best choice.
  • Tell the owner that feeling doubt regarding a euthanasia decision is natural and list the reasons why euthanasia is your best recommendation.
A

Answer: Assure the owner that feeling doubt regarding a euthanasia decision is normal and encourage the client to complete a quality-of-life score for the patient.

Explanation
Doubt is a normal part of the emotional response to considering humane euthanasia. Reviewing quality of life indicators can help clients feel more confident and experience less doubt regarding euthanasia decisions. Feeling doubt surrounding humane euthanasia is normal. When doubt is encountered, encouraging the owner to evaluate their own feelings surrounding euthanasia is preferred. While providing a clinical opinion about the appropriateness of humane euthanasia is helpful, the veterinarian should avoid pressuring the owner into performing humane euthanasia.

Encouraging a dialogue with the owner regarding the pet’s quality of life is more helpful in managing client doubts than listing clinical evidence in support of euthanasia.

Source: Handbook of Veterinary Communication Skills. Edited by Carol Gray and Jenny Moffett. Wiley-Blackwell, 2010. Pages 69-72.

The most appropriate response is to assure the owner that feeling doubt about a euthanasia decision is normal and encourage the client to complete a quality-of-life score for the patient. Doubt is a common emotional response, and reviewing quality of life indicators can help the owner make a more confident decision without feeling pressured. Engaging in a dialogue about the pet’s quality of life is more supportive than listing clinical reasons for euthanasia.

How well did you know this?
1
Not at all
2
3
4
5
Perfectly
32
Q

Dr. Santos diagnoses a patient with an emerging disease. There has been some speculation in the veterinary community that Substance A might be useful for treating this new disease. Dr. Santos treats the patient with Substance A, and the patient makes a full recovery. Dr. Santos is excited about Substance A and wants to let the veterinary community know about the case’s success. She decides to write a case report for publication in a major veterinary journal. Which of the following statements most accurately describes the scientific usefulness of the case report?

  • The results are as scientifically useful as those from a controlled clinical trial.
  • The results generate a hypothesis which can be tested in further studies.
  • The results confirm the hypothesis that Substance A is an effective treatment.
  • The results show that Substance A caused improvement in the patient’s symptoms.
A

Answer: The results generate a hypothesis which can be tested in further studies

Explanation
The purpose of a case study is to generate a hypothesis. Case studies cannot be used to test a hypothesis. Further research is needed to test the hypothesis that Substance A has a clinical benefit.

Case studies carry less strength of evidence than clinical trials. Case reports are not used to test hypotheses. A hypothesis may be generated in a case report, but it cannot be confirmed without further study. A cause-and-effect relationship cannot be established in a case report because there is no control group.

Source: (1) Reviewing the Evidence: How to be a discriminating Reader. Litster, A. L. American College of Veterinary Internal Medicine

(2) Conference Proceedings, 2017. (2) Guideline on writing a case report. Alsaywid, B. S. and Abdulhaq,
N. M. Urology annals, 2019, 11(2), pages 126-131.

Case reports are valuable for generating hypotheses, but they cannot test or confirm a hypothesis on their own. They provide preliminary observations that need to be investigated through controlled studies to establish a cause-and-effect relationship and determine the efficacy of treatments like Substance A.

How well did you know this?
1
Not at all
2
3
4
5
Perfectly
33
Q

Dr. Jenkins is reviewing lab work results for a 10-year-old female spayed Domestic Shorthair cat named Flopsy. Flopsy was presented yesterday for a routine wellness examination. Dr. Jenkins noted that the patient had lost 2.5 pounds since the previous wellness visit 12 months ago. Flopsy’s owner mentioned that she had been vomiting occasionally and drinking more water recently, but she was eating well and otherwise seemed to feel great. Results show the total T4 measurement is 7 pg/dL. Which of the following is the most appropriate option for discussing treatment with the owner?

  • Hyperthyroidism is usually treated with a medication called methimazole. It can be given orally or applied to the ear. There are several other options that we can discuss if the medication doesn’t work well for Flopsy.
  • We have a lot of options for treatment of hyperthyroidism. I will send comprehensive information via e-mail that should help you decide which treatment is best for Flopsy. If you have any questions after reading the information, please give me a call.
  • There are several options for treatment of hyperthyroidism. I will review the options with you today, and I would like to e-mail some more detailed information for you to review before you decide which you would like to use for Flopsy.
  • The best option for treatment of hyperthyroidism is radioactive iodine therapy, but most people don’t choose this option because it is very expensive. So, we will start a medication called methimazole and see how Flopsy responds.
A

Answer: There are several options for treatment of hyperthyroidism. I will review the options with you today, and I would like to e-mail some more detailed information for you to review before you decide which you would like to use for Flopsy.

Explanation
In this option, complex information is presented in small, manageable chunks. The client is receiving initial information directly from the veterinarian and can also review the written information at their own pace. This option involves repetition, which is helpful in reinforcing the information.

Finally, this answer incorporates both verbal and written communication to aid understanding.

While methimazole or radioiodine therapy are acceptable treatment options for hyperthyroidism, in each of these scenarios the veterinarian is making assumptions about the client’s goals. In the option that addresses methimazole therapy, the veterinarian has not adequately reviewed the other therapeutic options and has therefore not established informed consent. In the option that addresses radioactive iodine therapy, the veterinarian assumes that the client does not wish to incur the expense of therapy and leads the client toward therapy with methimazole instead.

While providing comprehensive information via e-mail is a wonderful idea, a combination of verbal and written communication is preferred.

Source: Handbook of Veterinary Communication Skills. Edited by Carol Gray and Jenny Moffett.
Wiley-Blackwell, 2010. Page 32-33.

The most appropriate option is to inform the owner that there are several treatment options for hyperthyroidism and to review these options together during the visit. Additionally, offering to e-mail detailed information allows the owner to review the options at their own pace before deciding on the best treatment for Flopsy. This approach ensures informed consent and combines verbal and written communication, which reinforces understanding and supports the decision-making process.

How well did you know this?
1
Not at all
2
3
4
5
Perfectly
34
Q

George is a long-term client who presents a deceased Golden Retriever for private cremation. The dog belonged to George’s mother, who recently suffered a stroke. George’s mother lived several states away at the time of the stroke, but she was moved into an assisted living facility near George’s home this week. George had been caring for the dog for the past two days until a more permanent placement could be made. However, the dog bolted out through the front door this morning and was hit by a car. George is visibly upset and repeatedly says hes not sure what he’s going to do. Which of the following is the most appropriate response?

  • Recognize that the client is experiencing a secondary loss and conclude that they are not in need of contact information for pet bereavement services.
  • Recognize that the client is experiencing a primary loss and suggest they perform an internet search for a pet loss support group.
  • Recognize that the client is experiencing an ambiguous loss and provide contact information for a virtual pet bereavement group.
  • Recognize that the client is experiencing a symbolic loss and provide contact information for a therapist experienced in pet bereavement counseling.
A

Answer: Recognize that the client is experiencing a symbolic loss and provide contact information for a therapist experienced in pet bereavement counseling.

Explanation
The best answer involves recognizing that the client is experiencing a symbolic loss. Symbolic pet losses occur when companion animals represent another loss in a client’s life. Providing contact information for a therapist experienced in pet bereavement counseling is a good choice for ensuring that the client is adequately supported.

Primary loss is felt following the death of a client’s personal pet. Secondary losses include the disruptions to the life of the client that are created by the primary loss. Ambiguous loss is experienced when the client does not know the fate of their companion animal, such as in cases of lost pets.

When a client experiences any type of pet loss, providing contact information for pet bereavement services is recommended. Having the information on-hand and readily available to clients is important.

Source: Handbook of Veterinary Communication Skills. Edited by Carol Gray and Jenny Moffett. Wiley-Blackwell, 2010. Pages 68-69 and 95-98.

How well did you know this?
1
Not at all
2
3
4
5
Perfectly
35
Q

A 12 year old Yorkie presents for euthanasia due to various medical problems and you have never seen the dog before. It is a walk-in appointment, as they were unable to be seen by their regular vet today. The dog is clearly in poor condition. The man presenting the dog states that this is his mother’s dog and she is not able to bring the dog herself. What would be the best option of the choices below?

  • Have the medical records faxed from the regular vet so that you can confirm the history and conditions for why the dog is presenting for euthanasia, then perform the procedure after the man signs a euthanasia request form.
  • Euthanize the dog as long as the man signs the consent form, because the dog is in poor condition and this is the most humane option.
  • Do not euthanize the pet since you have never seen this pet before and the owner is not present. The owner will need to be present to sign the authorization.
  • Call the owner directly to confirm her wishes for euthanasia and have another employee confirm over the phone as an additional witness. Have the agent sign the euthanasia request form.
A

Answer: Call the owner directly to confirm her wishes for euthanasia and have another employee confirm over the phone as an additional witness. Have the agent sign the euthanasia request form.

Explanation
This is a tricky situation and can be difficult to handle when the client’s emotions are involved.
Clearly the dog is in poor condition and the request for euthanasia is not unreasonable.

However, the owner needs to give direct permission for this to occur. Calling the owner for consent for the euthanasia is the correct way to approach this case. If the owner cannot be reached and there is no documentation for consent, then it is best to wait until there is valid documentation

If a euthanasia appointment is scheduled in advance and they are new clients, have them bring the medical records and ensure you will have the appropriate documentation you will need to carry out the euthanasia to try and avoid this type of conflict. When they schedule the appointment, if the owner will not be present, let them know confirmation by the owner over the phone or a written statement will be needed and should be brought and kept in the medical record.

Failure to receive such consent from the dog’s owner is a serious liability for you as the veterinarian. Despite the appearance, this man could be bringing the dog in for euthanasia against the dog’s owner’s wishes and it is your responsibility to properly document that euthanasia was requested by the owner.

The best option is to call the owner directly to confirm her wishes for euthanasia and have another employee confirm the conversation over the phone as an additional witness. The agent should then sign the euthanasia request form. This ensures that the owner’s consent is properly obtained and documented, which is crucial for legal and ethical reasons. It also helps to prevent any potential misunderstandings or conflicts, ensuring that the decision is made in accordance with the owner’s wishes. If the owner cannot be reached, it would be best to wait until proper documentation is obtained.

36
Q

Dr. Thomas is evaluating an 8-year-old female spayed Domestic Shorthair cat named Cutie. Cutie has been urinating outside of the litterbox and has foul-smelling urine. She has a history of Stage Il renal disease and has had urinary tract infections previously. In-house urinalysis fails to reveal bacteriuria, but the urinary pH is 8 and the urine specific gravity is 1.010. Dr. Thomas recommends a urine culture. Which of the following is the most accurate concerning informed consent for urine collection via cystocentesis?

-Dr. Thomas should perform the cystocentesis without obtaining verbal consent, since verbal consent is not necessary for routine procedures and a sterile sample is necessary.

-Dr. Thomas should describe the cystocentesis procedure to the owner, explain that a sterile urine sample is needed, and obtain written consent for the procedure.

-Dr. Thomas should discuss the cystocentesis procedure, explain that risks like bladder rupture and vagal response are uncommon, and obtain verbal consent for the procedure.

-Dr. Thomas should explain the cystocentesis procedure and assure the owner that the patient will not have any side effects, then obtain verbal consent for the procedure.

A

Answer: Dr. Thomas should discuss the cystocentesis procedure, explain that risks like bladder rupture and vagal response are uncommon, and obtain verbal consent for the procedure.

Explanation
The correct answer involves explaining the procedure, outlining the potential risks, and obtaining consent for the procedure. Verbal consent is typically adequate for a routine procedure when there is a long-standing veterinary-client-patient relationship. Informed consent is needed even for routine procedures.

Providing assurances that the procedures will go smoothly is inappropriate, as the owner should be made aware of potential complications as a part of informed consent. The risks of performing a procedure should be neither over- nor under-stated. Either written or verbal consent would be acceptable to document informed consent in this situation. However, in the answer option that includes written consent, the veterinarian failed to discuss the potential complications of the procedure.

Source: Handbook of Veterinary Communication Skills. Edited by Carol Gray and Jenny Moffett. Wiley-Blackwell, 2010. Pages 47-58.

The most accurate approach for obtaining informed consent for urine collection via cystocentesis is for Dr. Thomas to discuss the cystocentesis procedure, explain the potential risks—such as bladder rupture and vagal response, although these are uncommon—and obtain verbal consent. Verbal consent is typically sufficient for routine procedures when there is an established veterinary-client-patient relationship. It’s important to ensure the owner is aware of potential complications as part of the informed consent process, without overstating or understating the risks.

37
Q

Dr. Blackwell’s patient Snickers is a five-year-old male castrated Labrador Retriever mix who has been displaying a toe-touching lameness of the right hindlimb for one week. The clinical signs began suddenly after the pet yelped during agility class. The patient weighs 45 pounds. On physical examination, Dr. Blackwell notes a cranial drawer sign in the right hindlimb. Which of the following is the most appropriate way for Dr. Blackwell to discuss the options for management of an ACL tear in a dog patient?

  • Surgery is generally the best option for treatment of an ACL tear, and most clients choose this option even though the cost is high.
  • Conservative therapy is the option clients usually choose for treatment of an ACL tear because of the high cost of surgery.
  • Surgery is the generally the best option for treatment of an ACL tear, but most clients decline surgery for their pet due to the high cost.
  • Surgery and conservative therapy are the two main options for treatment of an ACL tear, and they vary in cost and effectiveness.
A

Answer: Surgery and conservative therapy are the two main options for treatment of an ACL tear, and they vary in cost and effectiveness.

Explanation
In this option, the veterinarian acknowledges that there is more than one treatment option and is honest about the cost differences without leading the client to a particular decision. The incorrect options each lead the client toward a particular choice. In each of the incorrect options, the veterinarian makes a judgement about the client’s finances and goals.

Source: Handbook of Veterinary Communication Skills. Edited by Carol Gray and Jenny Moffett.
Wiley-Blackwell, 2010. Pages 112-115.

The most appropriate way for Dr. Blackwell to discuss the management options for an ACL tear in Snickers is to explain that surgery and conservative therapy are the two main options, highlighting that they vary in cost and effectiveness. This approach acknowledges the different treatment possibilities without making assumptions about the client’s finances or goals, allowing the owner to make an informed decision based on their preferences and circumstances.

38
Q

Buck is an 8 year old male castrated Labrador Retriever who presents for a wellness visit. During examination, the prescapular and popliteal lymph nodes are found to be enlarged, firm and non-painful. Pathologist review of a fine needle aspirate from the lymph nodes supports a diagnosis of lymphoma. The clients are concerned about the costs of treating Buck’s illness. A nearby veterinary teaching hospital is currently enrolling newly diagnosed canine lymphoma patients in a clinical trial. The trial covers the cost of the testing and treatment that the enrolled patients receive. Which of the following is the most appropriate next step?

  • Decline to provide further treatment without the patient’s participation in the clinical trial.
  • Contact the teaching hospital to confirm the patient’s participation in the clinical trial.
  • Submit the paperwork and medical records required for the patient’s participation in the clinical trial.
  • Obtain informed consent from the client regarding the patient’s participation in the clinical trial.
A

Answer: Obtain informed consent from the client regarding the patient’s participation in the clinical trial.

Explanation
Obtaining informed consent from the client is the most appropriate next step. Informed consent is necessary for participation in clinical trials.

Participation in clinical trials must be voluntary and non-coercive. Owner consent should be established prior to discussing the patient’s case or sharing medical records with another veterinarian.

Source: Handbook of Veterinary Communication Skills. Edited by Carol Gray and Jenny Moffett. Wiley-Blackwell, 2010. Page 58.

The most appropriate next step is to obtain informed consent from the client regarding Buck’s participation in the clinical trial. Informed consent is essential for participation in clinical trials and ensures that the client’s decision is voluntary and non-coercive. Consent should be obtained before discussing the case or sharing medical records with another veterinarian.

39
Q

The role of a veterinarian in client communication and decision making is an important aspect of professional interaction. Three “styles” of communication have been described for such interactions:
Guardian (Paternalistic)
Teacher (Consultant)
Collaborator (Shared)
Which of these statements correctly describes how the style of a collaborator differs from the other 2 styles?

-In client communication, a veterinarian with a collaborator style is the primary decision maker whereas a veterinarian with a teacher or guardian style has the client be the primary decision maker
-In client communication, a veterinarian with a collaborator style presents all options with equal weight regardless of their treatment preference whereas a veterinarian with a teacher or guardian style does indicate their preference for treatment
-In client communication, a veterinarian with a collaborator style allows the client to be the primary decision maker whereas a veterinarian with a teacher or guardian style is the primary decision maker
-In client communication, a veterinarian with a collaborator style does not dominate the conversation while the veterinarian with either the guardian or teacher style does dominate the conversation

A

Answer: In client communication, a veterinarian with a collaborator style does not dominate the conversation while the veterinarian with either the guardian or teacher style does dominate the conversation

Explanation
The guardian style of veterinarian communicates in the following manner: They dominate the conversation and provide all of the information to the client. They typically present only the treatment option they feel is best and are therefore, the primary decision maker.

The teacher style of veterinarian provides information to the client but may also encourage them to seek additional information on their own. They typically dominate the conversation and present all options without emphasis toward the veterinarian’s own treatment preference.

The collaborator style of veterinarian shares decision making power and is approximately equal to the client in terms of doing most of the talking. They share medical information with the client that is relevant to the preferences provided by the client. All options are given and the client’s and veterinarian’s preferences are shared until a collaborative decision is made.

Any individual veterinarian can take on multiple roles depending on the situation and client but in general, the collaborator is viewed as the optimal choice for both veterinarian and client in the majority of situations.

In client communication, a veterinarian with a collaborator style differs from the guardian and teacher styles by not dominating the conversation. Instead, the collaborator shares decision-making power and engages in a balanced dialogue with the client. The guardian and teacher styles typically involve the veterinarian dominating the conversation, with the guardian making decisions and the teacher presenting all options without a preference. The collaborator is generally considered the optimal style, fostering a shared decision-making process based on mutual input.

40
Q

A new client has presented a sixteen-week-old German Shepherd puppy for evaluation. The puppy has been vomiting and having diarrhea for three days. The client has had the puppy for three weeks. The puppy is unwilling to eat or drink. The client has been using a syringe to give the puppy water and food, but the puppy vomits afterward. The owner also administered an unknown type and amount of “worm medicine” from the local feed store, but the puppy vomited after administration. Which of the following questions is the most appropriate for obtaining additional medical history for the patient?

  • Is the puppy up-to-date on its vaccines?
  • When and where was the puppy vaccinated?
  • Has the puppy had all of its vaccines?
  • Did the breeder give the puppy its vaccines?
A

Answer: When and where was the puppy vaccinated?

Explanation
“When and where was the puppy vaccinated?” is the most appropriate phrasing because the question is open-ended. The use of open-ended questions is encouraged to improve client communication. Open-ended questions allow the client to provide a comprehensive answer, which gives the most information to the medical team. Additionally, this option does not lead the client toward a particular answer. Questions beginning with “did,” “has”” and “is”” are closed questions, or questions for which there is only one answer (yes or no). Closed questions limit the information the client is being invited to provide. Additionally, the phrasing of the three incorrect options might lead the client to answer “yes”” even if they aren’t sure about vaccinations.

Source: Handbook of Veterinary Communication Skills. Edited by Carol Gray and Jenny Moffett. Wiley-Blackwell, 2010. Page 14.

The most appropriate question to obtain additional medical history is, “When and where was the puppy vaccinated?” This open-ended question encourages the client to provide detailed information without leading them toward a specific answer. Open-ended questions are effective in improving client communication, allowing the medical team to gather more comprehensive information about the patient’s history. In contrast, closed questions, which often result in simple yes or no answers, limit the information provided.

41
Q

Which of these represents an open-ended question that a veterinarian with good communication skills might use at the beginning of an appointment?

-When did your pet’s vomiting start?
-You seem very concerned about your pet’s condition.
-Why didn’t you bring your pet to the veterinarian when this first began last week?
-It sounds like your pet has been vomiting intermittently for several months.
-What’s been going on with your pet?

A

Answer: What’s been going on with your pet?

Explanation
What’s been going on with your pet? This is an example of an open-ended question that allows a client to tell their story in their own words without leading or prompting.

Why didn’t you bring your pet to the veterinarian when this first began last week? Questions beginning with why should be avoided as they imply guilt or responsibility and may invoke a defensive response from a client.

It sounds like your pet has been vomiting intermittently for several months. This is an example of a reflective listening statement. While this can be very effective in communication, it is not an open-ended question.
When did your pet’s vomiting start? This is a specific, pointed question rather than an open-ended one that will likely invoke a short response. This may allow the discovery of important information but should be reserved until after a client has explained the relevant history in their own words.

You seem very concerned about your pet’s condition. This is an example of an empathic statement that is also used by veterinarians with good communication skills but it is not an open-ended question.

The best example of an open-ended question that a veterinarian with good communication skills might use at the beginning of an appointment is, “What’s been going on with your pet?” This question invites the client to share their story in their own words without leading or prompting, allowing for a more comprehensive understanding of the situation. Open-ended questions like this one are crucial for gathering detailed information and building rapport with the client.

42
Q

Dr. Blackman is running late for an appointment with Snickers and her owner Janice. About an hour ago, another patient was presented for emergency evaluation after being hit by a car. That patient required immediate stabilization and significant monitoring. Janice has been waiting for Dr. Blackman for about thirty minutes and has complained to the reception staff several times. When Dr. Blackman goes into the exam room for the appointment, what is the most effective way to dissipate Janice’s anger about the wait?

  • We are sorry that you were kept waiting. Emergencies must come first.
  • I’m sorry that you were kept waiting. We have been handling an emergency.
  • We are sorry that you had a long wait. We use a triage system for emergencies.
  • I apologize for your wait. I was delayed by an emergency with another patient.
A

Answer: I apologize for your wait. I was delayed by an emergency with another patient.

Explanation
When clients are angry due to being kept waiting, personal ownership of an apology (“I apologize…”) is preferred over an apology without ownership (“I’m sorry that…”). Additionally, apologies which were accompanied by an explanation for the wait are preferred. In each of the incorrect options, there was no direct ownership of the apology. While triage of emergencies is an important clinic policy, relating this information to an owner is unlikely to dissipate their anger regarding their wait.

Source: Handbook of Veterinary Communication Skills. Edited by Carol Gray and Jenny Moffett. Wiley-Blackwell, 2010. Page 117-118.

The most effective way to dissipate Janice’s anger about the wait is to say, “I apologize for your wait. I was delayed by an emergency with another patient.” Taking personal ownership of the apology and providing a brief explanation for the delay acknowledges the inconvenience and helps to diffuse the client’s frustration more effectively than simply stating a policy or offering an apology without ownership.

43
Q

Marvin, a six-year-old male castrated Great Dane, has been presented to the local emergency clinic due to a one-hour history of a distended abdomen. The patient is retching repeatedly. STAT radiographs of the abdomen reveal a gas-distended stomach and a classic “double bubble” radiographic sign. The veterinarian on call recognizes this as a case of gastric dilatation-volvulus. What are the most appropriate steps for the veterinarian to take in establishing informed consent for medical stabilization and surgery?

-Explain that medical stabilization and emergency surgery is indicated, review possible complications with the owner, and obtain verbal and written consent for the necessary procedures.
-Educate the owner regarding the need for medical stabilization and emergency surgery, review the survival rates established in the scientific literature for this condition, and obtain verbal consent for the procedures.
-Explain that medical stabilization and emergency surgery is indicated, assure them that everything will turn out fine, and obtain verbal and written consent for the necessary procedures.
-Discuss the need for medical stabilization and emergency surgery, warn the owner that the pet is not likely to survive the procedures, and obtain written consent for the procedures.

A

Answer: Explain that medical stabilization and emergency surgery is indicated, review possible complications with the owner, and obtain verbal and written consent for the necessary procedures.

Explanation
Explain that medical stabilization and emergency surgery is indicated, review possible complications with the owner, and obtain verbal and written consent for the necessary procedures. The risks of performing a procedure should be neither over- nor under-stated. Obtaining both verbal and written consent for procedures is preferred. Providing assurances that the procedures will go smoothly is inappropriate, as the owner should be made aware of potential complications as a part of informed consent. While presenting death as a potential complication is reasonable, the risks should not be overstated. Other potential surgical and anesthetic complications should also be discussed. Obtaining both verbal and written consent for surgical procedures is preferred.

Source: Handbook of Veterinary Communication Skills. Edited by Carol Gray and Jenny Moffett. Wiley-Blackwell, 2010. Pages 47-58.

The most appropriate steps for the veterinarian are to explain that medical stabilization and emergency surgery are indicated, review possible complications with the owner, and obtain both verbal and written consent for the necessary procedures. It’s important to provide a balanced explanation of the risks without overstating or understating them, ensuring the owner is fully informed. This approach helps the owner understand the situation and the potential outcomes, leading to a more confident and informed decision.

44
Q

Dr. Jones is examining Snuggles, an 8-year-old male castrated Domestic Shorthair cat. Snuggles’ owner is Anna, who is a physician. Anna noted that Snuggles wasn’t eating well two days ago. Today Snuggles began vocalizing in the litterbox. Snuggles lives with two other cats, so Anna is not sure whether he is urinating successfully. On palpation of the abdomen, Dr. Jones feels a large, turgid urinary bladder. Which of the following is the most appropriate way for Dr. Jones to begin a discussion about urethral obstruction in cats?

  • Do you already know about urethral obstruction in cats?
  • Most clients aren’t familiar with urethral obstruction in cats.
  • I’m sure you know all about urethral obstruction in cats.
  • How familiar are you with urethral obstruction in cats?
A

Answer: How familiar are you with urethral obstruction in cats?

Explanation
Use of a question to gauge the client’s current knowledge base is recommended. This option does not suggest an assumption of the client’s level of knowledge. This option is posed as an open question, which maximizes the amount of information that the client is likely to share. The use of open-ended questions is encouraged to improve client communication. Open-ended questions allow the client to provide a comprehensive answer, which gives the most information to the medical team. Additionally, this option does not lead the client toward a particular answer. Questions beginning with “do” are closed questions, or questions for which there is only one answer (yes or no). Closed questions limit the information the client is being invited to provide. Both statement options assume the level of knowledge the client possesses and should therefore be avoided.

Source: Handbook of Veterinary Communication Skills. Edited by Carol Gray and Jenny Moffett. Wiley-Blackwell, 2010. Page 14 and 31-33.

The most appropriate way for Dr. Jones to begin a discussion about urethral obstruction in cats is to ask, “How familiar are you with urethral obstruction in cats?” This open-ended question helps gauge the client’s current knowledge without making assumptions. It encourages Anna to share what she knows, allowing Dr. Jones to tailor the explanation to her level of understanding. Using open-ended questions like this one is recommended for effective client communication.

45
Q

Mia, a veterinary assistant, has been working with Dr. Sandoval for the past six months to improve her anesthetic monitoring skills. While Dr. Sandoval was performing surgery today, Mia alerted her to the patient’s low blood pressure readings. Dr. Sandoval then walked Mia through the hypotension checklist, and Mia was able to adjust the inhalant anesthetic concentration and increase the IV fluid rate to resolve the patient’s blood pressure issues. Which of the following is the most appropriate option for providing positive reinforcement and feedback for Mia?

  • I can tell you have been working really hard!
  • You have really been doing a good job lately!
  • Your performance at work has been improving lately!
  • Great job recognizing hypotension in our patient today!
A

Answer: Great job recognizing hypotension in our patient today!

Explanation
This feedback is both specific and timely. Providing detailed feedback is recommended rather than using general statements. Incorrect options are those that use general statements instead of specific feedback.

Source: 101 Veterinary Practice Management Questions Answered. Donnelly, A. L. 2010. American Animal Hospital Association. Page 105.

The most appropriate option for providing positive reinforcement and feedback to Mia is: “Great job recognizing hypotension in our patient today!” This feedback is specific and timely, which is more effective than general statements in reinforcing her skills and encouraging continued learning and development.

46
Q

Dr. Tremblay is walking through the reception area when he witnesses an interaction between a client and the head receptionist. The client has presented her pet Willie for a wellness visit. The receptionist does not greet the client or smile. During the check-in process, the receptionist remains seated, fails to make eye contact with the client, and speaks with an exasperated tone. Dr. Tremblay is not satisfied with the welcome the client received. Which of the following is the most appropriate option for providing feedback to the receptionist?

  • Wait one month, until the receptionist’s next official review, to address the client interaction.
  • Tell the receptionist that she needs to be friendly even if she doesn’t feel cheerful while at work.
  • Discuss the importance of open body language and a warm demeanor when interacting with clients.
  • Let the receptionist know that clients can tell when staff members are in an irritable mood.
A

Answer: Discuss the importance of open body language and a warm demeanor when interacting with clients.

Explanation
When giving feedback, the focus should be on correcting behavior. Feedback should also be specific and timely.

Feedback should not focus on intangible qualities such as the employees perceived attitude or intentions.

Provide feedback that is both specific and timely.

Source: 101 Veterinary Practice Management Questions Answered. Donnelly, A. L. 2010. American Animal Hospital Association. Page 105.

The most appropriate option for providing feedback to the receptionist is to discuss the importance of open body language and a warm demeanor when interacting with clients. This approach focuses on specific, correctable behaviors and provides timely feedback that can help improve the receptionist’s interactions with clients, enhancing the overall client experience.

47
Q

Zelda has owned a goat farm and creamery for three months. The previous owner no longer had the time to invest in the business, and Zelda purchased the farm at well below market rate. She still has a job in the city as an accountant, but she would like for the creamery to eventually be her primary source of income. Zelda does not have any previous experience with goats, but she is excited about the venture and wants to ensure the goats are cared for appropriately. She has formed a strong emotional bond with several of the goats. Recently Zelda has been noticing that some of her favorite goats have diarrhea and don’t seem to feel well. Dr. Harper confirms infection with Haemonchus contortus. Several of the goats have high FAMACHA scores. Which of the following is the most appropriate option for educating the owner about genetic selection for Haemonchus resistance?

  • Dr. Harper tells Zelda that the goats are infected with the barber pole worm, administers an anthelmintic to all of the affected goats, assures the owner that the goats will be feeling much better soon, and mentions that sometimes genetic selection is used to improve herd health.
  • Dr. Harper explains that the goats are infected with Haemonchus contortus, tells the owner to administer an anthelmintic, advises the owner to research genetic selection for Haemonchus resistance, and recommends that the owner read a textbook and several journal articles regarding Haemonchus management.
  • Dr. Harper explains that the goats have an intestinal parasite called the barber pole worm, reviews the FAMACHA scale with the owner using non-technical language, and cautions the owner that culling may need to be considered for goats who repeatedly have high FAMACHA scores.
  • Dr. Harper tells Zelda that the goats are infected with an intestinal parasite, administers an anthelmintic to all of the goats with high FAMACHA scores, and decides that discussion of genetic selection and possible culling is not necessary since the owner has formed a strong bond with the goats.
A

Answer: Dr. Harper explains that the goats have an intestinal parasite called the barber pole worm, reviews the FAMACHA scale with the owner using non-technical language, and cautions the owner that culling may need to be considered for goats who repeatedly have high FAMACHA scores.

Explanation
Diagnosis, prognosis and treatment options should be communicated in clear, non-technical language that the owner can easily understand. Providing the owner with easy-to-understand information that is designed for use by non-experts, such as the FAMACHA scale, is recommended. The prognosis of an illness should not be over-or under-stated. Presenting a realistic picture of the prognosis regarding herd health is important. Assuring the owner that the goats will recover is inappropriate. Discussion of standard herd management options including culling should not be omitted from a discussion about herd health and management options, even when the owner has a strong bond with animals in a production setting. The use of highly technical medical terminology should also be avoided when communicating with clients. Use of clear, common language is recommended. It is best to select appropriate educational resources that reflect the knowledge level of the client. While recommending textbooks and journal articles to highly knowledgeable and experienced goat producers might be a reasonable step, this owner’s low experience and training level would benefit more from non-technical resources designed for lay people. Instructing an inexperienced owner to administer an anthelmintic with no further direction is not ideal.

Source: Handbook of Veterinary Communication Skills. Edited by Carol Gray and Jenny Moffett. Wiley-Blackwell, 2010. Pages 25-38.

The most appropriate option for educating Zelda about genetic selection for Haemonchus resistance is for Dr. Harper to explain that the goats are infected with the barber pole worm, review the FAMACHA scale using non-technical language, and caution Zelda that culling may need to be considered for goats with repeatedly high FAMACHA scores. This approach ensures that Zelda understands the situation clearly, including the importance of herd management practices like culling, while using language that is accessible to someone with her level of experience. Providing realistic information about the prognosis and management options is crucial, especially given Zelda’s strong emotional bond with her goats and her goal of making the creamery her primary source of income.

48
Q

Anna is the owner of a twenty-five-year-old mare who has been declining for quite a while. Anna has been monitoring the mares quality of life, and she is concerned that it is no longer adequate. Anna is anxious and fearful about the idea of humane euthanasia. She schedules a pre-euthanasia appointment with her veterinarian to get more information. When discussing the process of humane euthanasia, which of the following options represents the most appropriate phrasing?

  • Explain that the injection will stop the heart and cause death.
  • Explain that the injection will cause the pet to go to sleep forever.
  • Explain that the injection will cause the pet to pass away.
  • Explain that the injection will create cardiac and respiratory arrest.
A

Answer: Explain that the injection will stop the heart and cause death.

Explanation
This explanation is clear and accurate. This phrasing also contains non-technical language that is easy for a lay person to understand. When communicating with clients, avoid using euphemisms such as “pass away,” or “sleep forever.” Using overly technical medical terminology should also be avoided when communicating with clients, as some clients may not understand what “cardiac arrest” means.

Source: Handbook of Veterinary Communication Skills. Edited by Carol Gray and Jenny Moffett. Wiley-Blackwell, 2010. Page 100-109.

The most appropriate phrasing when discussing the process of humane euthanasia with Anna is to explain that “the injection will stop the heart and cause death.” This explanation is clear, accurate, and uses non-technical language that is easy for a layperson to understand. Avoiding euphemisms and overly technical terms ensures that the information is communicated in a straightforward and compassionate manner.

49
Q

Dr. Jackson is seeing one of her favorite patients today. Tippy is a thirteen-year-old Border Collie with a history of mitral valve disease and syncope. Tippy has seen a veterinary cardiologist regularly for years, but at the last visit the cardiologist indicated that Tippy has end-stage disease. The cardiologist could make no recommendations for additional medication. The cardiologist’s recommendation was to continue supportive care until quality of life becomes unacceptable. Tippy’s owner has scheduled a consultation today because Tippy has had one or more syncopal episodes daily for the past three days. What is the most appropriate question for the Dr. Jackson to use to initiate a quality-of-life discussion with Tippy’s owner?

-Do you want to give Tippy some more time with you?
-What are your goals regarding end-of-life care for Tippy?
-Do you think that it’s time to let Tippy go?
-Is humane euthanasia your plan for today’s visit?

A

Answer: What are your goals regarding end-of-life care for Tippy?

Explanation
This is the most appropriate phrasing because the question is open-ended. The use of open-ended questions is encouraged to improve client communication. Open-ended questions allow the client to provide a comprehensive answer, which gives the most information to the medical team. Additionally, this option does not lead the client toward a particular answer. All of the incorrect options feature closed questions. Questions beginning with “do,” and “is” are closed questions, or questions for which there is only one answer (yes or no).

Closed questions limit the information the client is being invited to provide. When communicating with clients, avoid using euphemisms such as “let the patient go.” Finally, veterinarians should avoid leading the owner toward a particular decision.

Source: Handbook of Veterinary Communication Skills. Edited by Carol Gray and Jenny Moffett. Wiley-Blackwell, 2010. Page 14 and 100-126.

The most appropriate question for Dr. Jackson to initiate a quality-of-life discussion with Tippy’s owner is, “What are your goals regarding end-of-life care for Tippy?” This open-ended question encourages the owner to share their thoughts and priorities, providing valuable information for the medical team. It avoids leading the client toward a specific answer and fosters a more meaningful and comprehensive discussion about Tippy’s care.

50
Q

Sissy is a fourteen-year-old female spayed Domestic Shorthair cat who has been chronically vomiting for the past month. Dr. Smith notes a mass in the cranial abdomen on palpation. After several appointments to discuss the mass, the owner declines referral for ultrasound and exploratory surgery with a specialist. Instead, the owner chooses exploratory surgery with Dr. Smith. Before surgery, the owner explains that she intends to proceed with humane euthanasia if the mass is inoperable and that she does not wish to be present if euthanasia occurs. In surgery, Dr. Smith identifies a large, inoperable mass near the pylorus. Based on this information, what is the most appropriate way for Dr. Smith to proceed?

-Proceed with euthanasia based on the owner’s previous statements regarding her wishes. Since the possibility of euthanasia was previously discussed, further consent and documentation is not necessary.
-Obtain verbal consent for euthanasia via phone, with a second staff member listening to and confirming the owner’s consent, then document the conversation in the medical record before proceeding with euthanasia.
-Obtain verbal consent via phone, then have the owner return to the clinic to sign a written consent form prior to euthanasia.
-Accept verbal consent for euthanasia via phone, proceed with euthanasia, and have the owner mail in a written consent form.

A

Answer: Obtain verbal consent for euthanasia via phone, with a second staff member listening to and confirming the owner’s consent, then document the conversation in the medical record before proceeding with euthanasia.

Explanation
For cases involving a previously known client and patient, in which the possibility of euthanasia is discussed in advance of an anesthetic procedure, verbal consent via phone is acceptable. Having more than one staff member confirm the consent for euthanasia via phone and document the conversation is recommended. In such a case, written consent is not essential, and insisting on written consent may create more stress for the client. When accepting consent for euthanasia via phone, having a second staff member confirm the client’s wishes is recommended. If verbal consent for euthanasia has been given and the procedure has already been completed, then written consent is unnecessary. Having the owner return to the clinic to sign a written consent form may create more stress for the client. However, some type of owner consent is required prior to humane euthanasia. Consent prior to euthanasia is always necessary.

Source: Handbook of Veterinary Communication Skills. Edited by Carol Gray and Jenny Moffett. Wiley-
Blackwell, 2010. Pages 55-56.

The most appropriate way for Dr. Smith to proceed is to obtain verbal consent for euthanasia via phone, with a second staff member listening to and confirming the owner’s consent. This conversation should then be documented in the medical record before proceeding with euthanasia. This approach ensures that consent is properly obtained and recorded, reducing stress for the client while adhering to ethical and legal standards.

51
Q

Rebecca is a long-term client who presented her twelve-year-old Golden Retriever for evaluation after the patient collapsed while on a walk. Physical examination reveals a very pale mucus membrane color and a distended abdomen with a fluid wave. Abdominocentesis yields >20 mL of blood with a hematocrit higher than that of the peripheral blood, confirming the presence of a hemoabdomen. What is the most appropriate approach for communicating these findings to the owner?

-Explain that you found blood in the abdominal cavity, then discuss the prognosis and treatment options using pleasant, innocuous language.
-Explain that the patient is bleeding into the abdominal cavity, then discuss the prognosis and treatment options using positive, upbeat language.
-Explain that the patient has a bleeding mass in the abdomen, then discuss the prognosis and treatment options using advanced medical terminology.
-Explain that you have some bad news to share, then discuss the clinical findings, prognosis, and treatment options using clear, non-technical language.

A

Answer: Explain that you have some bad news to share, then discuss the clinical findings, prognosis, and treatment options using clear, non-technical language.

Explanation
Using a verbal cue to prepare the owner for bad news is recommended. Findings, prognosis and treatment options should be communicated in clear, direct language that the owner can easily understand. The use of euphemisms or overly positive language should be avoided when relating bad news to clients. Presenting a realistic picture of the prognosis is important. The use of highly technical medical terminology should be avoided when communicating with clients. Though a bleeding mass in the abdomen is a common cause of hemoabdomen in dogs, we do not yet have enough information to present this differential as a definitive diagnosis.

Source: Handbook of Veterinary Communication Skills. Edited by Carol Gray and Jenny Moffett. Wiley-Blackwell, 2010. Page 104.

The most appropriate approach is to begin by gently preparing the owner by saying, “I have some bad news to share.” Then, discuss the clinical findings, prognosis, and treatment options using clear, non-technical language. This approach ensures that Rebecca is prepared for the difficult news and that the information is communicated in a way she can easily understand. Avoiding euphemisms or overly technical terms helps to present a realistic and compassionate explanation of the situation.

52
Q

Which of the following is not true about Hemophilia A?

  • Treatment of choice to stop a bleeding event is fresh plasma or fresh frozen plasma
  • It is transmitted by an X-linked pattern of inheritance so females are less likely affected
  • It is caused by a deficiency in Factor VIII of the clotting cascade
  • Affected animals have a normal activated clotting time and partial thromboplastin time
A

Answer: Affected animals have a normal activated clotting time and partial thromboplastin time

Explanation
The correct answer is affected animals have a normal activated clotting time and partial thromboplastin time.

Affected animals usually have less than 10% of Factor VIll and have prolonged ACT and APTT. The disease commonly affects dogs and cats and mostly only males, since it is an X-linked recessive trait. Females with the gene for the disease are usually unaffected carriers unless they were the offspring of an affected male and a carrier female.

***PowerPage: Hemostasis

In Hemophilia A, affected animals do not have normal activated clotting time (ACT) and partial thromboplastin time (APTT). Instead, these times are prolonged due to less than 10% of Factor VIII. This X-linked recessive disease commonly affects male dogs and cats, while females are typically unaffected carriers unless bred from an affected male and carrier female.

** Coagulation Protein Disorders in Animals: Key Information**

Definitions and Types:

•	Congenital Coagulation Disorders: Early-age symptoms, often fatal with severe deficiencies.
•	Factor I (Fibrinogen) Deficiency: Severe bleeding; treated with fresh-frozen plasma.
•	Factor II (Prothrombin) Disorders: Rare; treated with plasma or whole blood.
•	Factor VII Deficiency: Prolonged PT; minimal clinical bleeding.
•	Factor VIII (Hemophilia A): Common in dogs, X-linked; treated with cryoprecipitate.
•	Factor IX (Hemophilia B): Similar to Hemophilia A; treated with plasma.

Acquired Coagulation Disorders:

•	Liver Disease: Reduced production of coagulation proteins, potential DIC.
•	Vitamin K Deficiency: Impaired production of Factors II, VII, IX, X.
•	Anticoagulant Rodenticide Toxicity: Inhibits vitamin K recycling; treated with vitamin K1.

Disseminated Intravascular Coagulation (DIC):

•	Causes: Secondary to infections, heat stroke, neoplasia.
•	Stages: Hypercoagulable (initial), hypocoagulable (advanced).
•	Diagnosis: Thromboelastography, hemostasis tests (APTT, PT, fibrinogen).
•	Treatment: Address underlying disease, heparin for hypercoagulable stage, plasma for hypocoagulable stage.

https://www.merckvetmanual.com/circulatory-system/hemostatic-disorders/coagulation-protein-disorders-in-animals

53
Q

Heparin is frequently used as an anticoagulant in veterinary medicine. Heparin’s mechanism of action is related to which of the following?

  • Binds to antithrombin, which subsequently accelerates the interaction between antithrombin and activated factor X or thrombin
  • Binds to calcium, which is necessary for the coagulation cascade to proceed
  • Promotes the conversion of plasminogen to plasmin
  • Binds to and inhibits factors II, VIl and IX
A

Answer: Binds to antithrombin, which subsequently accelerates the interaction between antithrombin and activated factor X or thrombin

Explanation
Heparin inactivates activated factor X and thrombin, thus halting the coagulation cascade. Remember, factor X is related to the common pathway of the coagulation cascade, whereas thrombin is necessary to convert fibrinogen to fibrin.

Heparin works as an anticoagulant by binding to antithrombin, which then accelerates the inhibition of activated factor X and thrombin. This action halts the coagulation cascade, as factor X is part of the common pathway, and thrombin is crucial for converting fibrinogen to fibrin.

54
Q

Blood lactate is a commonly measured parameter in clinical practice as a reflection of anaerobic metabolism. Which of the following is an end-product of glycolysis and is also converted to lactate under anaerobic conditions?

  • Succinate
  • Glucose 6-phosphate
  • Pyruvate
  • Citrate
  • Acetyl coenzyme-A (acetyl Co-A)
A

Answer: Pyruvate

Explanation
During the anaerobic process of glycolysis, a single molecule of glucose forms 2 molecules of pyruvate in the cell cytoplasm. Under aerobic conditions, pyruvate is converted to acetyl coenzyme-A and fed into the TCA cycle. In anaerobic conditions, pyruvate is preferentially transformed to lactate in an effort to maintain ATP production and allow regeneration of NAD (required for continued glycolysis).

Pyruvate is the end-product of glycolysis and is converted to lactate under anaerobic conditions. In the absence of oxygen, this conversion helps maintain ATP production and regenerates NAD, essential for continued glycolysis. Under aerobic conditions, pyruvate is converted to acetyl coenzyme-A for the TCA cycle.

55
Q

In health, lactate is primarily metabolized by what organ and should measure less than what value (mmol/L)?

  • Liver; less than 2.5 mmol/L
  • Kidney; less than 5 mmol/L
  • Lung; less than 6.5 mmol/L
  • Liver; less than 5 mmol/L
  • Pancreas; less than 6.5 mmol/L
  • Kidney; less than 2.5 mmol/L
A

Answer: Liver; less than 2.5 mmol/L

Explanation
Lactate is primarily metabolized by the liver and, in health, it should be low (< 2.5 mmol/L). The kidneys and skeletal muscle also metabolize lactate to a lesser extent. Lactate is typically associated with anaerobic metabolism and has been used to predict survival in gastric dilatation volvulus in dogs and colic and sepsis in horses.

56
Q

The synthesis of which of these clotting factors is dependent on vitamin K?

  • Factor IX
  • Factor III
  • Factor V
  • Factor VIII
  • Factor VI
A

Answer: Factor IX

Explanation
The correct answer is factor IX. The vitamin K dependant factors are II, VII, IX, and X. This is important when animals are exposed to a vitamin K antagonist, such as many rodenticides.

***PowerPage: Hemostasis

57
Q

When reading a bacterial culture and sensitivity report, which antibiotic is used to indicate methicillin resistance?

  • Cefazolin
  • Penicillin
  • Rifampin
  • Oxacillin
  • Erythromycin
A

Answer: Oxacillin

Explanation
Oxacillin is the preferred antimicrobial used to test for methicillin resistance. Oxacillin resistance implies resistance to all penicillins, cephalosporins, imipenem, and beta lactam/beta lactamase inhibitors and combinations such as amoxicillin/clavulanic acid. Oxacillin is used instead of methicillin because it is more stable and widely available.

Oxacillin is used to test for methicillin resistance in bacterial culture and sensitivity reports. Oxacillin resistance indicates resistance to all penicillins, cephalosporins, imipenem, and beta-lactam/beta-lactamase inhibitor combinations like amoxicillin/clavulanic acid. It is preferred over methicillin due to its stability and availability.

58
Q

Vascular ring anomalies such as persistent right aortic arch commonly cause what clinical sign in young affected dogs?

  • Hyperactivity
  • Dyspnea
  • Regurgitation
  • Hemorrhage
A

Answer: Regurgitation

Explanation
The correct answer is regurgitation. Vascular ring anomalies entrap the esophagus and trachea.

Circular compression of the esophagus results in physical obstruction and regurgitation. Less commonly, they affect the trachea.

***PowerLecture: Vascular Ring Anomalies

59
Q

You diagnose a dog with a larger left to right ventricular septal defect with advanced sequelae to the abnormality. Which of the following is the most likely radiographic appearance of this dog’s thorax?

  • Left atrial and left ventricular enlargement with pulmonary hyperperfusion
  • Right ventricular and left atrial enlargement with pulmonary vasculature hypoperfusion
  • Right ventricular enlargement with pulmonary hyperperfusion and a normal left heart
  • Enlargement of all 4 heart chambers and hyperperfusion of the pulmonary vasculature
A

Answer: Left atrial and left ventricular enlargement with pulmonary hyperperfusion

Explanation
The correct answer is left atrial and left ventricular enlargement with pulmonary hyperperfusion. In dogs with a VSD, the shunt occurs during systole when both ventricles are contracting so blood is shunted from the left ventricle almost directly into the pulmonary artery. The right ventricle acts as a conduit between the left ventricle and pulmonary artery and is less commonly affected by the shunted blood. The excess blood flow is found in the pulmonary vasculature, left atrium, and left ventricle, thus enlarging these structures.

60
Q

A 6-month old male Newfoundland presents for a physical exam. Cardiac auscultation reveals a 3/6 left systolic murmur at the heart base. Further diagnostics reveal the diagnosis of mild to moderate subaortic stenosis. What should you tell the owner?

  • The prognosis is fair. Sudden death may occur at any time, and the dog has an increased risk for infective endocarditis.
  • The prognosis is good if the dog has not developed congestive heart failure yet. Prophylactic treatment with furosemide for congestive heart failure should be prescribed.
  • The prognosis is poor. Most cases of mild to moderate aortic stenosis do not live past 4 years of age despite all therapy.
  • The prognosis is relatively good. Balloon dilation of the stenotic region is the treatment of choice and has a high success rate.
A

Answer: The prognosis is fair. Sudden death may occur at any time, and the dog has an increased risk for infective endocarditis.

Explanation
The correct answer is the prognosis is fair. For dogs with very mild subaortic stenosis, the prognosis is good. Dogs with moderate or severe disease, however have a much more guarded prognosis.

Sudden death may occur at any time, and the dog has an increased risk for infective endocarditis.
Ventricular arrhythmias can occur and may be the factor leading to sudden death in these patients.
Animals with aortic stenosis are at increased risk for infective endocarditis and should be placed on prophylactic antibiotics whenever they undergo a surgical procedure, dental procedure, get an open wound, or experience anything that may potentially lead to bacteremia. Balloon dilation does not improve the prognosis for dogs with subaortic stenosis, though does improve the prognosis for dogs with pulmonic stenosis. Starting furosemide before the onset of congestive heart failure is not advised.

61
Q

As shown in the photo below, cardiac hemangiosarcoma in dogs is most commonly found on what area of the heart?

  • Left atrium
  • Left ventricle
  • Right atrium
  • Right ventricle
A

Answer: Right atrium

Explanation
The correct answer is right atrium. Although the reason for this predilection is unknown, this is the most common site in the heart for hemangiosarcoma to occur.

***PowerPage: Hemangiosarcoma

62
Q

In macaques, this relatively non-pathogenic disease can cause a fatal encephalitis if transmitted to humans.

  • Tuberculosis
  • Hepatitis B
  • Herpes simiae
  • Measles
A

Answer: Herpes simiae

Explanation
The correct answer is Herpes simiae. This is a herpes B virus. Old world monkeys can transmit this disease by bite or scratch. Macaques are the most frequent carriers. In monkeys, the disease can cause mild cold sore like lesions in times of stress. This strain of herpes is potentially fatal to humans. Similarly, humans carry a strain of herpes (herpes simplex virus) which can cause fatal encephalitis in monkeys. Hepatitis B, tuberculosis, and measles are all zoonotic, but do not cause encephalitis.

63
Q

A 16 week-old spayed female kitten is presented to you for diarrhea. The owners note that the kitten has had diarrhea since they adopted her from the shelter 2 weeks ago. The kitten has paperwork showing that she was dewormed twice with Pyrantel prior to adoption. The owners are concerned because they have another cat and a 2 month-old baby at home. What should you tell the owners?

  • Coccidia are very easy to disinfect; however, it is easily shared from animals to humans and the owner should be concerned about the safety of the baby
  • Coccidia are incredibly environmentally resistant and difficult to disinfect; Coccidia are also species-specific so the baby is not at risk but the other cat could become infected
  • Coccidia are very easy to disinfect, making it simple to eradicate from the environment; there is no risk to anyone in the household
  • Coccidia are incredibly environmentally resistant and infectious across species; the owner should be worried for her other cat and baby
A

Answer: Coccidia are incredibly environmentally resistant and difficult to disinfect; Coccidia are also species-specific so the baby is not at risk but the other cat could become infected

Explanation
Coccidiosis or infection with the protozoan
Isospora is very common in young cats and dogs.
The disease is very difficult to disinfect and rid from the environment. It is important to pick up the stools as soon as possible to avoid sporulation and contamination of the environment with cysts.
Luckily, Isospora is species-specific so zoonosis is not a concern.

64
Q

A 2-year old indoor-outdoor female spayed domestic short hair cat presents to you for inappetance and vomiting. The cat vomits and the contents include several 3 cm long nematodes with spine-covered heads.
You are able to identify the worms as Gnathostoma.

Which of the following is true about these worms?

  • The parasite does not cause disease in humans
  • Proper disposal of feces eliminates the threat of transmission to other animals
  • The parasite is typically found in dry, dusty environments
  • The life cycle involves a rodent intermediate host
  • The life cycle involves a flea intermediate host
A

Answer: Proper disposal of feces eliminates the threat of transmission to other animals

Explanation
The life cycle of Gnathostoma involves a small copepod intermediate host (similar to Dracunculus worms). Copepods are small crustaceans that are found in freshwater habitats (i.e. ponds). Eggs passed in the feces are not infectious unless first ingested by copepods so proper disposal of feces prevents transmission to other animals.

Adult worms live in the mucosa of the stomach and cause gastritis. They can also create nodules in the stomach which can ulcerate and lead to severe peritonitis. Human cases of gnathostoma usually occur from ingestion of undercooked fish or other animals and can cause gastritis or peritonitis. Larva may undergo cutaneous or neural migration (rare).

Control is typically achieved by preventing cats from hunting in areas where the parasite is found.

It is thought that albendazole is effective against gnathostomiasis.

65
Q

What is the permanent dental formula for a cat?

  • 2(13/3 C1/1 P3/2 M1/1)
  • 2(13/3 C1/1 P2/3 M1/3)
  • 2(13/3 C1/1 P3/3 M1/3)
  • 2(13/3 C1/1 P4/4 M2/3)
A

Answer: 2(13/3 C1/1 P3/2 M1/1)

Explanation
The correct answer is 2(13/3 C1/1 P3/2 M1/1).

66
Q

A 2-year old male neutered indoor/outdoor Maine Coon presents for diarrhea for the past week. On exam he is bright, alert, and responsive. His temperature is 103.4 F (39.7 C), heart rate is 140 beats per minute, and respiration rate is 40 breaths per minute. Mucoid brown to green diarrhea is found on rectal examination. The cat is still eating well and has normal activity levels. Blood work shows a monocytosis and eosinophilia. On fecal exam you see tiny oocysts (see image). You should inform the owner of which of the following?

Lifelong signs may reoccur with flare ups in the future
Treatment will be long and the cat should be isolated
Treatment could have potential adverse ettects
Clean litterbox daily and wash hands thoroughly. Immunocompromised people should avoid the litter box.

A

Answer: Clean litterbox daily and wash hands thoroughly. Immunocompromised people should avoid the litter box.

Explanation
Toxoplasmosis is a zoonotic disease. Cats are the reservoir. Clinical signs are typically seen in younger cats but can occur in older cats or immunocompromised cats as well. Signs can vary from diarrhea to fever, malaise, orthopedic diseases, uveitis, or neurological signs. Oocysts are shed in the feces, where they take around 5 days to sporulate and become infective. Isolating the affected animal will decrease risk of transmission to other animals.
Cleaning the litterbox daily will decrease the risk of exposure of infective oocysts to humans.
Toxoplasmosis can cause birth defects or still births in humans, so pregnant women should avoid the litter box. Immunocompromised people should also avoid cleaning the litter box. Treatment of choice for cats is typically clindamycin.

67
Q

Your client raises show rabbits and her cat named Neffer has been eating the rabbit feces. She brings Neffer’s fecal matter in for examination and the fecal float reveals Eimeria (see image). Which of the following treatments are indicated for this cat?

  • Fenbendazole
  • Sulfasalazine
  • Doxycycline
  • Sulfadimethoxine
  • No treatment is necessary
A

Answer: No treatment is necessary

Explanation
This coccidian is not parasitic in dogs and cats and no treatment is necessary for the cat. The parasite has shown up on fecal exam because it is merely passing through the digestive tract secondary to coprophagy.
Eimeria is identified as having 4 sporocysts or a distinct micropyle cap. Eimeria are parasitic in birds, reptiles, and herbivores. The owner should be informed of the parasite, and her rabbits should be treated.
Isospora is the infectious coccidian in dogs and cats and is treated with Sulfadimethoxine (Albon).

68
Q

When performing a subtotal colectomy on a feline patient, what blood vessel limits the amount of colon that you are able to remove?

  • Caudal mesenteric artery
  • Pudendal artery
  • Left colic artery
  • Ileocolic artery
A

Answer: Ileocolic artery

Explanation
The correct answer is ileocolic artery. The site for colonic resection is limited by tension on the ileocolic artery when trying to suture your new end of colon to the rectum. Sometimes the tension is too great and instead of a colocolic anastomosis, an ileocolic anastomosis must be performed.

Essentially you are trying to connect a section of ascending colon to the rectum. Now that can be pretty far! Performing an ileocolic anastomosis is not ideal because you eliminate the ileocecal valve, and that may predispose the animal to bacterial overgrowth. The caudal mesenteric artery gives branches to the rectum and descending colon. The left colic artery also feeds the descending colon. The pudendal artery supplies the external genitalia. The ileocolic artery provides blood supply to the ascending and transverse colon.

69
Q

A 6-year old domestic short hair presents to you with a history of weight loss and steatorrhea. On examination, you can palpate thickened small intestinal loops of bowel. You are suspecting EPI (exocrine pancreatic insufficiency). How would you diagnose this condition?

  • Measure serum cobalamin concentration
  • Measure plasma cobalamin concentration
  • Measure serum fPLI concentration
  • Measure serum amylase and lipase concentrations
  • Measure serum fTLI concentration
A

Answer: Measure serum fTLI concentration

Explanation
EPI is characterized by the lack of production of pancreatic digestive enzymes and is best diagnosed by measuring trypsin-like immunoreactivity. EPI occurs when 90% of pancreatic exocrine function is destroyed. fTLI concentration is measured from a fasted blood sample and a trypsinogen level less than 8ug/L is diagnostic. Typically cats with EPI also have concurrent chronic pancreatitis as well as low levels of cobalamin. The exocrine pancreas is responsible for secreting intrinsic factor. Intrinsic factor binds cobalamin to allow for gut absorption.

Amylase and lipase concentrations do not provide reliable information regarding pancreatic function in cats.

70
Q

You are out in the field and you notice a horse tilt his head up and curl back his lips in a manner that makes it appear to be “grimacing”. You identify this as a flehmen response. What is thought to be happening when this response occurs?

  • Scents are moved to the vomeronasal organ
  • This is a sign of sexual interest or arousal and you need to be careful if the horse is nearby
  • This is a display of aggression or dominance that a horse will display when it detects the presence of other stallions
  • This is a sign of colic
  • This is a pathognomonic sign of yellow star thistle toxicity
A

Answer: Scents are moved to the vomeronasal organ

Explanation
The flehmen response, as described in the question, helps animals trap pheromone scents in the vomeronasal organs (VNOs) so they can be analyzed more closely. Pheromones are the chemical signals emanating from other animals.

When a horse draws in an organic odor, he curls up his lip to temporarily close the nasal passages and hold the particles inside. The upward head tilt seems to help the airborne molecules linger in the VNOs, which are located under the floor of the horse’s nasal cavity.

While sex pheromones are the most common flehmen trigger, they are not the only ones, and the response itself does not indicate sexual interest.

71
Q

A 4 year old Ayrshire cow presents with a two day history of depression, anorexia, fever, conscious proprioceptive deficits, circling, right sided head tilt, and head-pressing.
A neurologic exam identified cranial nerve deficits.
Particularly, the right ear is drooping, the right eye appears “dropped”, and she is drooling from the right side of her mouth. What is the most likely diagnosis?

  • Polioencephalomalacia
  • Mycoplasma bovis
  • Listeriosis
  • Perennial ryegrass staggers
A

Answer: Listeriosis

Explanation
Listeria monocytogenes may cause an acute meningoencephalitis with microabscesses in the area of the brainstem and cranial nerve roots.

However, there are other clinical forms of listeriosis that may be seen, such as abortion and septicemia of neonates. Usually, only individual animals are affected and not the herd. The animals commonly become infected after consuming spoiled silage. In general, the clinical signs seen are associated with dysfunction of the caudal brain stem, cerebellar peduncles, or spinal cord. Mycoplasma bovis will not result in central nervous system disease, especially in such a short time span (can cause otitis media-interna).

Polioencephalomalacia does not result in cranial nerve signs and is caused by a thiamine deficiency. Ingestion of perennial ryegrass (grows between June and September in northern hemisphere) will cause ataxia and tremors as a result of activating GABA receptors when it is infected with an endophytic fungus.

***PowerLecture: Central Nervous System

72
Q

In horses, this disease will result in an acute necrotizing hepatitis.

  • Theiler’s disease
  • Chronic-active hepatitis
  • Hyperadrenorticism
  • Pyrrolizidine alkaloid toxicity
A

Answer: Theiler’s disease

Explanation
The correct answer is Theiler’s disease (aka serum sickness or serum hepatitis). Theiler’s disease will cause an acute multifocal to diffuse hepatitis along with necrosis. PA toxicity results in the triad of biliary hyperplasia, fibrosis, and megalocytosis.

Chronic-active hepatitis can cause a suppurative or lymphoplasmacytic inflammatory response in the liver.

***PowerPage: Hepatobiliary Diseases of Horses

***PowerLecture: Hepatobiliary Disorders

73
Q

Average gestation length in the mare is how long?

  • 405 days
  • 345 days
  • 225 days
  • 305 days
  • 265 days
A

Answer: 345 days

Explanation
The correct answer is 345 days, although gestational length can range from 330-360 days.

It is good to have knowledge of previous gestational lengths because individual brood mares have the same/similar gestational lengths every year, allowing accurate estimate of foaling date. Additionally, this is important because many horse owners want mares to foal in mid-January for the reason that in racing, all horses have an artificial birthday on January 1st. In order to achieve this, it is recommended to breed in mid-February.

Since the horse is seasonally polyestrous and would naturally start cycling in April or May, artificial lighting is often used starting in mid-December to promote estrus in mid-February.

Most foals are considered premature if born prior to 320 days and prolonged gestation is typically associated with greater than 365 days.

***PowerLecture: Gestation and Parturition

74
Q

Severe combined immunodeficiency is a lethal autosomal recessive trait in Arabian foals. Heterozygotes are clinically normal. If the heterozygote carrier rate for the genetic mutation is 8%, what is the expected frequency of Arabian foals that are homozygous for the mutated allele?

  • 4%
  • 0.64%
  • 25%
  • 2%
  • 0.16%
  • 0.064%
A

Answer: 0.16%

Explanation
You do not need to know anything about the disease in this question in order to get the correct answer. You are told that the disease is recessive and has a carrier rate of 8%

Because the trait is recessive, homozygotes will be the only individuals affected. In order for a foal to be born homozygous for the trait, BOTH parents MUST be carriers. The chances of both parents being carriers is 8% × 8% (or 0.08 × 0.08) = 0.0064 or 0.64%. If both parents are carriers, the offspring has a 1 in 4 chance of inheriting two mutant alleles (50% chance for each allele from each parent).

Since the chances of both parents being carriers is 0.64% and the chance of having a homozygous offspring in that case is 1 in 4, the overall expected frequency of diseased foals is 0.0064 × 0.25= 0.0016 or 0.16%.

75
Q

A 12-year old indoor only Devon Rex presents with a 2 month history of weight loss, decreased appetite, and diarrhea with occasional vomiting. Bloodwork is unremarkable. The closest referral for ultrasound or endoscopy is 2 hours away and the owner is unable to drive for further diagnostics. You perform an abdominal exploratory and take biopsies of the stomach and intestine. The histopathology results show lymphocytic plasmacytic enteritis with a moderate number of eosinophils. Which of the following is the best recommendation for this cat?

  • Prednisolone and cisplatin for lymphoma
  • Fortiflora to restore normal intestinal bacteria
  • Metoclopramide to assist with Gl motility and decrease nausea
  • Hypoallergenic diet and prednisolone for inflammatory bowel disease
  • Fenbendazole to treat the likely occult Giardia infection
A

Answer: Hypoallergenic diet and prednisolone for inflammatory bowel disease

Explanation
Lymphocytic plasmacytic enteritis is consistent with inflammatory bowel disease. The main treatments of this disease include controlling the underlying cause for the disease, controlling inflammation, and controlling bacterial overgrowth when needed. Steroids and hypoallergenic diet are the mainstay treatments for this disease.

Prednisolone and budesonide are the two corticosteroids most often used. Metronidazole can also aid in treatment.

Cisplatin is a chemotherapeutic agent that is contraindicated for the use in cats for any disease.
Fortiflora is a probiotic that could possibly help with clinical signs of IBD, however is not necessarily a treatment for the disease itself.

Metoclopramide could potentially help clinical vomiting from underlying IBD, but is a pro-motility agent that could worsen diarrhea and is not a treatment for IBD.

While routine parasite control and fecal testing are important, fenbendazole is not a direct treatment for IBD. This indoor cat should probably have a fecal test, but Giardia is unlikely the cause for the histopathologic findings.

76
Q

A 2-year old male neutered domestic shorthair named Cooper presents for 2-month history of coughing with occasional wheezing. He is mildly tachypneic when he is active per the owner’s observation. Cooper lives inside only. Currently no tachypnea or labored breathing are appreciated. Heart and lungs auscult normally.

Bloodwork is unremarkable other than a mild eosinophilia. Heartworm and fecal testing are negative.
Chest radiographs reveal a mild interstitial pattern with diffuse bronchial wall thickening. Which diagnosis and therapy would be most appropriate?

  • Pneumonia, broad spectrum antibiotic
  • Congestive heart failure, furosemide
  • Airway collapase from trauma, no treatment indicated
  • Hypertension, amlodipine
  • Aelurostrongylus, fenbendazole
  • Feline asthma, prednisolone
A

Answer: Feline asthma, prednisolone

Explanation
Feline asthma, also known as feline allergic bronchitis, is acute or chronic airway inflammation that occurs due to various stimuli. The triggers of this airway inflammation are largely unknown but underlying environmental allergens are suspected.

The mainstays of therapy include corticosteroid administration and in some cases bronchodilators.
In this young indoor cat with a negative fecal and heartworm test, infectious causes are less likely.
Pneumonia typically causes more of a patchy alveolar pattern, most often causes a fever, and would be less likely a chronic condition.

Congestive heart failure in a young cat with no heart murmur is very unlikely.

77
Q

What is the most common reason for post-anesthetic cortical blindness in cats?

  • Gas anesthetic reaction
  • Arrhythmias
  • Hypoxia
  • Elevated end-tidal CO2
A

Answer: Hypoxia

Explanation
Post anesthetic cortical blindness results from poor perfusion during anesthesia. The visual cortex is extremely sensitive to the effects of hypoxia (poor perfusion/oxygenation) which can result in blindness. Not all arrhythmias will necessarily compromise tissue perfusion (for example isolated VPC’s or first degree AV block etc.).

Hypersensitivity or allergy to anesthetic gases used (such as isoflurane gas) are rarely documented. Elevated end-tidal CO2 indicates poor ventilation (such as from inadequate fresh gas flow, increased dead space, saturated up soda-lyme, a deep anesthetic plane, etc.), but not necessarily poor perfusion.

78
Q

A domestic short hair cat presents for decreased appetite due to chronic renal disease. Which of the following is an appropriate appetite stimulant?

  • Famotidine
  • Maropitant
  • Metoclopramide
  • Mirtazapine
  • Omeprazole
A

Answer: Mirtazapine

Explanation
Although, all of the options given can potentially help a cat with renal disease feel better, the only medication considered an appetite stimulant is mirtazapine. Mirtazapine is both a noradrenergic and serotonergic antagonist. The antagonism of presynaptic alpha-2 receptors increases the release of Noradrenaline and Serotonin (5HT). The blockage of the 5HT2A, 5HT2C and 5HT3 receptors redirects Serotonin to the 5HT1A receptor. Mirtazapine also antagonizes H1 receptors and the antagonism of this receptor and the 5HT2C receptor appear to be the mechanism by which appetite is increased.

Famotidine (Pepcid) is an H-2 receptor blocker while omeprazole (Prilosec) is a proton pump inhibitor. Both reduce gastric acid secretion.

Maropitant (Cerenia) is an anti-emetic. It is a neurokinin-1 inhibitor, which inhibits Substance P in the CNS.
Metoclopramide is a prokinetic and anti-emetic drug. The mechanism for its anti-emetic effects is from antagonism of dopamine at its receptors in the chemo-receptor trigger zone of the brain.

79
Q

In dogs, large doses of epinephrine cause which of the following?

  • Increase in total peripheral resistance, increase in cardiac contractility, bronchodilation
  • Increase in arterial blood pressure, decrease in cardiac contractility, bronchodilation
  • Decrease in total peripheral resistance, increase in cardiac contractility, bronchodilation
  • Increase in arterial blood pressure, increase in cardiac output, bronchoconstriction
A

Answer: Increase in total peripheral resistance, increase in cardiac contractility, bronchodilation

Explanation
The correct answer is increase in total peripheral resistance, increase in cardiac contractility, bronchodilation.

Epinephrine is an alpha- and beta-adrenergic stimulant which causes peripheral vasoconstriction, increase in total peripheral resistance, increase in cardiac contractility and cardiac output, and bronchodilation. It is often used in cardiac arrests and anaphylactic shock. It is also found in some local anesthetic agents because of its vasoconstrictive effects, which prolongs effect of the anesthetic.

80
Q

A 5-year old West Highland White Terrier presents for seborrhea oleosa and pruritus. You perform skin scrapes and a cytology and diagnose Malassezia dermatitis. Which of the following is the best treatment option?

  • Ketoconazole
  • Prednisone
  • Cephalexin
  • Carprofen
  • Griseofulvin
A

Answer: **

Explanation
Ketoconazole is the best choice. This medication, along with other -azoles (fluconazole, itraconzole), are commonly prescribed treatments for Malassezia. Terbinafine is another antifungal medication that is becoming more popular for the treatment of Malassezia and dermatophytosis. Although, Griseofulvin is an antifungal it is not effective against Malassezia. Prednisone and carprofen may help with inflammation, but not clear the infection.

81
Q

Which of the following is a component of nephrotic syndrome?

  • Hypocholesterolemia
  • Hypercoagulability
  • Hyperglobulinemia
  • Hypoproteinemia
A

Answer: **

Explanation
The correct answer is hypoproteinemia. The four components of nephrotic syndrome are proteinuria, hypoproteinemia, hypercholesterolemia, and ascites or edema. This syndrome occurs with protein-losing nephropathies such as glomerulonephritis or amyloidosis. Hypercoagulability is not a component of nephrotic syndrome, although it can occur with protein-losing nephropathies due to the loss of antithrombin Ill.

82
Q

A 2-year old male castrated Border Collie presents for a 1-week history of small bowel diarrhea. A fecal flotation shows numerous Giardia cysts. What is the treatment of choice for this dog?

  • Metronidazole
  • Albendazole
  • Decoquinate
  • Ipronidazole
A

Answer: metronidazol

Explanation
The correct answer is metronidazole. You should know that fenbendazole is actually the treatment of choice for Giardia, but metronidazole has historically been the most widely accepted option. If fenbendazole is not offered as an answer choice, choose metronidazole. If fenbendazole were offered as an answer choice, it would be the best option.

83
Q

Which of the following is not a treatment option for a ruptured cranial cruciate ligament for a dog?

  • Recession trochleoplasty
  • Leveling of the tibial plateau
  • Cranial transposition of the fibular head
  • Weight reduction
A

Answer: Recession trochleoplasty

Explanation
The correct answer is recession trochleoplasty. This technique is part of a surgical repair method for luxating patellas. Weight reduction is nearly always a therapeutic component to managing orthopedic disease. Leveling of the tibial plateau and cranial transposition of the fibular head are two surgical options for correcting a torn cranial cruciate ligament.

***PowerPage: Cruciate Ligament Disease

***PowerLecture: Cranial Cruciate Ligament Rupture

84
Q

You have a 6-year old female spayed Cocker Spaniel with immune mediated hemolytic anemia (IMHA), and would like to start her on azathioprine and prednisone. Which one of the following is NOT a reported possible side effect of azathioprine?

  • Gastrointestinal upset
  • Nephrotoxicity
  • Bone marrow suppression
  • Acute pancreatitis
  • Hepatotoxicity
A

Answer: Nephrotoxicity

Explanation
Nephrotoxicity is not a reported side effect of azathioprine. All others on this list have been reported as side effects or have been associated with the use of azathioprine. Gl upset is the most common of the 4 listed, although it is still relatively infrequent. The other 3 listed side effects are uncommon, but can be life-threatening, so they must be discussed with the owner before starting treatment.

Cats are more sensitive to these side effects than dogs because of a deficiency in thiopurine methyltransferase, the enzyme used to metabolize azathioprine. Azathioprine should be used with caution, if at all, in cats.

85
Q

Which of the following tests can be used to differentiate pituitary-dependent hyperadrenocorticism from an adrenal tumor causing hyperadrenocorticism in dogs?

  • ACTH stimulation test
  • Low-dose dexamethasone suppression test
  • Urine cortisol:creatinine
  • Resting cortisol level
A

Answer: Low-dose dexamethasone suppression test

Explanation
The correct answer is low-dose dexamethasone suppression test. Tests used to try to differentiate PDH from an adrenal tumor include LDDST, HDDST, abdominal ultrasound, and endogenous plasma ACTH assay. With hyperadrenocorticism patients, LDDST and HDDST results show elevated cortisol levels at the 8-hour post-dexamethasone administration sample (normal animals would show suppressed cortisol levels at the 8-hour sample). PDH patients will show brief cortisol suppression at the 4-hour post-dexamethasone sample on some LDDST and HDDST tests. Patients with adrenal tumors causing Cushing’s will not exhibit suppression of cortisol levels at the 4-hour post-sample.

Although no patients with adrenal tumors should show suppression at the 4-hour post-sample, some PDH patients will not show cortisol suppression at the 4-hour post-sample either. Just as a recap, you look at the 8-hour post-dexamethasone sample to determine if the patient has Cushing’s or not (should be elevated with Cushing’s), and you look at the 4-hour post-sample to try and differentiate if the patient has PDH (will show cortisol suppression) or if you cannot determine if the patient has PDH or an adrenal tumor (no cortisol suppression).

86
Q

A dog presents to your clinic for fever and lymphadenopathy. You perform an aspirate of the lymph node and see large, bipolar staining coccobacilli. The owner mentions that he saw his dog eating a rat a few days ago. What is causing the dog’s illness?

  • Yersinia pestis
  • Pasteurella multocida
  • Staphylococcus aureus
  • Francisella tularensis
A

Answer: Yersinia pestis

Explanation
The correct answer is Yersinia pestis. In order to make this determination remember that Yersinia pestis is usually transmitted to cats and dogs as a result of ingesting infected rodents or via bites from the prey’s fleas. Dog and cat fleas are poor vectors of Plague. Dogs usually recover and you may lance the “buboes” and flush it, but dispose of organic material properly.